You are on page 1of 234
PEARSON EDEXCEL INTERNATIONAL A LEVEL aus ‘atshes by Posen Edation Line, 80 Star, Landon, WO2R ORL. wwipersongboaechoolcom ‘Copies ool speciteabon ora Pearson qualifications ay be found on the ‘webate: hipe//quatteatons pearon.com “ext © Poaroon Eavoston Lites 2018 ted ny Ere Pract Designed by © Pearzon Eaueation Lets 2018 “ypest by TochSet La, Gstesheas, UK (rig uations © Pausan Eduction Lined 2018 ‘stood by © Tech Sat tc, Gatesnead, UK ‘Cove design by © Pearson Eacatan Lito 2018 “ho rights of Gog Atwood, Jack Baraloun, lan Betisen, Gordon Das, Ket Gales, Dania Glebe, Asta Machewon, Arne MeAloe Bronwen Man ‘5 Neholon, Dane Diver oe Pea, Keth Pledger. Cong San, Joe Serko, ay Sth, Gao Staley, Robert Ward Pony and Oave Whikns obscene a5 he autorsof the werk have boon aseotd by hem n accordance wit he ‘Copyright, Designs and Patents Act 1988, Fret pba 2018 10987654921 ‘rsh Library Cataloguing n Pubcon Data ‘Nostlogie recor forte book aval fom th teh Ubrary Copyright noice [Anahi reserve. No pa ofthis publication ray be reproduced in ary frm or by {ary means (heucing phoxecopyng or stn in ary medium by econ means {nd hater or not anion riety to sore oor us of his pubicater) ‘uttrout he ween porrisson ofthe copy une, except accordance wih the prouons of he Copyright. Design and Pater Act 088 of under the rs ‘fa toorcoseued my the Copyrant Loaner Agere, amar on 06 Fer {apa London, ECAR TEN (uiula.coul. Appicatne fore copyright owners \writon permission shou be aderessod to the pubes Prete by Neogatia in Stovaka Plotre Croce ‘The blr would het thank the flowing for he Kd permission to reproduce thr petooraphe: ‘Nomy Stock Phot: Xa 8; Goty Image: Grok 150, Henalon and Sees 18, ‘rwaron ; NABA: 170 Science Photo Librery Lit CMS EXPERIMENT, CER; ‘Shutterstock com: Dain M85, Dae 159 tt 8 Cover images: Fro: Getty Images: Wear Vin Sin ‘rete rant cover: Shutterstock com: Dry Labarow Allothr images © Pearson Educator ‘larwork © Peston Edveaton Endorsement Statement rower to enaure hal hi retcuce fe highly sia forthe atocited Pearson auton, thas bob trou ave process by the awaring body “Ths proces covfme al thwsoure fy comes the teaching a earn contnt ofthe sposteaton or prof a soctoation at which ti med lso onfima that emonavates an appropiate balance betweon the devaonent fai sts role an ncetndig aan panton Ensocstmant a nt cover any gusance on atetementactitas or proceenea {@9-procice uestons or advice on how to wsewerasossrien questions) Iruted mtn recite, rer does t pesca parc appreach 10h teaching oF delivery ofa elated course. le the pubiars have made erry arp to enue that advice onthe (qualflcaton ands assessments accurate, the oficial epecfeaten ahd ‘icoited assesment gute mater re the any eave sere Irtoraton ana stous aways be refs to for dott guidance, Pearson examiners have nt contributed to any actions inthis resource relevant to ‘ramnaton papers er whien hey have responsbiy. ascecoment st by Poaron. Endorsmant of arovoure dees mean that he resource i rogue to schave ths Pearson quailtin, pox dows rea that Is tre ery eutabe matora avaible to support the qualcaton and ary rsource 1s produced by te awarding body shal icudo tha and ther approprate COL COURSE STRUCTURE ABOUT THIS BOOK vi QUALIFICATION AND ASSESSMENT OVERVIEW viii EXTRA ONLINE CONTENT x 1 ALGEBRAIC EXPRESSIONS 1 2 QUADRATICS 18 3 EQUATIONS AND INEQUALITIES 36 4 GRAPHS AND TRANSFORMATIONS 57 REVIEW EXERCISE 1 81 5 STRAIGHT LINE GRAPHS 85 6 TRIGONOMETRIC RATIOS 104 7 RADIANS 133 8 DIFFERENTIATION 150 9 INTEGRATION 170 REVIEW EXERCISE 2 181 EXAM PRACTICE 185 GLOSSARY 187 ANSWERS 190 INDEX 222 aca CHAPTER 1 ALGEBRAIC CHAPTER 4 GRAPHS AND EXPRESSIONS 1 TRANSFORMATIONS 57 1.1 INDEX LAWS 2 4.1 CUBIC GRAPHS 58 1.2 EXPANDING BRACKETS, 4 4,2 RECIPROCAL GRAPHS 62 1.3 FACTORISING 6 4.3 POINTS OF INTERSECTION 63 1.4 NEGATIVE AND FRACTIONAL 4.4 TRANSLATING GRAPHS 67 INDICES 9 4.5 STRETCHING GRAPHS a 1.5 SURDS 12 4.6 TRANSFORMING FUNCTIONS 75 1.6 RATIONALISING DENOMINATORS 13 CHAPTER REVIEW 4 78 CHAPTER REVIEW 1 15 REVIEW EXERCISE 1 81 CHAPTER 2 QUADRATICS 18 2.1 SOLVING QUADRATIC EQUATIONS §= 19 2.2 COMPLETING THE SQUARE 22 CHARTER 2S) RAIGHT EINE 2.3 FUNCTIONS 25 ‘GRAPHS 85 2.4 QUADRATIC GRAPHS g Slyemete a os me visennaman so 52 EQUATIONS OF STRAIGHT LINES 99 Peepetaeaicatin 535.3 PARALLEL AND PERPENDICULAR LINES 93 5.4 LENGTH AND AREA 96 CHAPTER 3 EQUATIONS AND CHAPTER REVIEW 5 99 INEQUALITIES 36 eos a CHAPTER 6 TRIGONOMETRIC 3.2 QUADRATIC SIMULTANEOUS RATIOS 104 calimmonis gg _ 8-1 THE COSINE RULE 105 3.3 SIMULTANEOUS EQUATIONS ON 6.2 THE SINE RULE 10 GRAPHS 40 6.3 AREAS OF TRIANGLES 116 3.4 LINEAR INEQUALITIES a4 6.4 SOLVING TRIANGLE PROBLEMS 118 3.5 QUADRATIC INEQUALITIES 4g 8-5 GRAPHS OF SINE, COSINE AND 3.6 INEQUALITIES ON GRAPHS 49 meen 1 3.7 REGIONS 51 6.6 TRANSFORMING TRIGONOMETRIC CHAPTER REVIEW 3 54 GRAPHS 125 CHAPTER REVIEW 6 129 CUTS at CHAPTER 7 RADIANS 133 7.1 RADIAN MEASURE 134 7.2 ARC LENGTH 135, 7.3 AREAS OF SECTORS AND SEGMENTS 139 CHAPTER REVIEW 7 145 CHAPTER 8 DIFFERENTIATION 150 8.1 GRADIENTS OF CURVES. 151 8,2 FINDING THE DERIVATIVE 154 8.3 DIFFERENTIATING x" 157, 8.4 DIFFERENTIATING QUADRATICS 159. 8.5 DIFFERENTIATING FUNCTIONS ‘WITH TWO OR MORE TERMS: 161 8.6 GRADIENTS, TANGENTS AND NORMALS 163 8.7 SECOND ORDER DERIVATIVES 165 CHAPTER REVIEW 8 167 CHAPTER 9INTEGRATION 170 9.1 INTEGRATING x" 1 9.2 INDEFINITE INTEGRALS 173 9.3 FINDING FUNCTIONS: 176 CHAPTER REVIEW 9 178 REVIEW EXERCISE 2 181 PCS 4 ABOUT THIS BOOK The following three themes have been fully integrated throughout the Pearson Edexcel International Advanced Level in Mathematics series, so they can be applied alongside your learning, 1. Mathematical argument, language and proof + Rigorous and consistent approach throughout, + Notation boxes explain key mathematical language and symbols 2. Mathematical problem-solving ‘The Mathematical Problem-Solving Cycle + Hundreds of problem-solving questions, fully integrated r pei the problem 1 into the main exercises + Problem-solving boxes provide tips and strategies interpret results sees einen = Challenge questions provide extra stretch t j proces and 3. Transferable skills tepeent information = + Transferable skills are embedded throughout this book, in the exercises and in some examples + These skills are signposted to show students which skills they are using and developing Finding your way around the book Each chapter starts with Ist f Laarang objectives The Prior knowledge check helps make sure you ae ready to start the chapter Glossary terms wil be Identified by bold blue tox, on hel frst appearance Each chapters mapped tothe specification content or e295. rotenone “The eal world applications of the maths you ae about fe lean are highlighted atthe start ofthe ‘chapter CUCL Slop bystep worked ‘examples focus en the key types of questions you'lneed te tack. Tiansieabee sue ave signposted whore they naturaly occur in tho exercises and examples rercise questions are caretuly graded ‘so tey increase in csc ane gracuaty bring you up 0 exam stancor races we packed ‘ath exam-ayle {Questions to encure you Bre ready forthe eras roblen-solving boxes provide hints, tips and sratogles, and Watcn out boxes highight areas where students dften ose marks in ther exams Jqueations Exam-style questions are flagged with © Problem-solving ‘questions are flagged Each chaptor onde with a Chapter reviow with © ‘anda Summay of ey points Yor every fw chapters, a Review exercise fee file rr ere —< on of exaresiye ueations Exam practice Mathematis International Advanced Subsidiary! ‘Advanced Level Pure Mathematics 1 Review exercise’ 1 ‘ul practice paper at the back of the bbook sips you prepare forthe real thing vil vili QUALIFICATION AND ASSESSMENT OVERVIEW QUALIFICATION AND ASSESSMENT OVERVIEW Qualification and content overview Pure Mathematics 1 (P1) is a compulsory unit in the following qualifications: International Advanced Subsidiary in Mathematics International Advanced Subsidiary in Pure Mathematics International Advanced Level in Mathematics International Advanced Level in Pure Mathematics Assessment overview The following table gives an overview of the assessment for this unit. We recommend that you study this information closely to help ensure that you are fully prepared for this course and know exactly what to expect in the assessment, aC Availability Pl:PureMathematics1 —|33}%0f1AS—|75 ‘Thour30mins | January, June and October PapercodeWMAIVOL | 162 of AL Fist assessment january 2019 IAS - international Advanced Subsidiary IAL - International Advanced A Level Assessment objectives and weightings cr ee reer Recall, select and use their knowledge of mathematical facts, concepts and techniques in a 20% | variety of contexts. 30% Construct rigorous mathematical arguments and proofs through use of precise statements, logical deduction and inference and by the manipulation of mathematical expressions, including the construction of extended arguments for handling substantial problems presented in unstructured form, Recall, select and use their knowledge of standard mathematical models to represent ‘x03. | situations in the real world; recognise and understand given representations involving standard models; present and interpret results from such models in terms of the orginal situation, including discussion of the assumptions made and refinement of such models. Comprehend translations of common realistic contexts into mathematic; use the results of ‘804 | calculations to make predictions, or comment on the context; and, where appropriate, read 5% critically and comprehend longer mathematical arguments or examples of applications. Use contemporary calculator technology and other permitted resources (such as formulae 805. | booklets or statistical tables) accurately and efficiently; understand when not to use such 5% ‘technology, and its limitations, Give answers to appropriate accuracy. 30% 10% CEU Uu cassie My a Ld Relationship of assessment objectives to units ed oy Marks out of 75 30235 25-30 15 510 “5 % ue Bho 20 a3} ihe Calculators Students may use a calculator in assessments for these qualifications. Centres are responsible for making sure that calculators used by their students meet the requirements outlined below. Students are expected to have available a calculator with at least the following keys: +,-, +, 7,32, ve, 4», ln x, e%, x1, sine, cosine and tangent and their inverses in degrees and decimals of a degree, and in radians; memory. Prohibitions Calculators with any of the following facilities are prohibited in all examinations: + databanks + retrieval of text or formulae + built-in symbolic algebra manipulations + symbolic differentiation and/or integration + language translators + communication with other machines or the internet un ey @ Extra online content Whenever you see an Online box, it means that there is extra online content available to support you. SolutionBank SolutionBank provides worked solutions for questions in the book. Download all the solutions as a PDF or quickly find the solution you need online, Use of technology Explore topics in more detail, visualise problems and consolidate your understanding, Use pre-made GeoGebra activities or Casio resources for a graphic calculator. EDD ins nest ofnerecion GP EY graphically using technology. GeeGebra GeoGebra-powered interactives CASIO. Graphic calculator interactives o_tex Interact with the maths you are learning using GeoGebra’s easy-to-use tools Explore the maths you are learning and gain confidence in using a graphic calculator Calculator tutorials Our helpful video tutorials will guide you through how to use your calculator in the exams. They cover both Casio's scientific and colour graphic calculators. CED Wea cuteness sep git autoinsrctons ‘the "C; and power functions on your calculator. Finding the value of the first derivative te acess the fanction press ‘on exactly which buttons to press and what should appear on your calculator’s screen 1 ALGEBRAIC + smpleting this chapt Multiply and divide in Expand a single term over brackets and collect like terms > pages 2-4 Expand the product of two or three expressions > pages 4-6 Fa linear, quadratic and simple cubic expressions > pages 6-9 ind use the laws of indices + pages9-12 Simplify and use the rules of surds “pages 12-13 Ration: > pages 13-15 2nen + mn = 3m Sx+243x2-Tx-12 ‘+ International GCSE Mathematics er of 2: b 26322 « International GCSE Mathematles Expand: a 3(r+4) © 6(2x-5y) «International GCSE Mather Write down the highest common factor of: a 2and16 b 6xand 8x 3xy © International 20x ie 24 < 40x ‘€ International GCSE Mathematics S ientists use indices y large numb A quantum computer with 1000 qubits (quantum bits lues simultaneously. This is reater than the number of particles in the observable un ae Gs} & Index laws = You can use the laws of i + a xatsane + an"sarzan—" + @ysam This isthe index, is power or exponent. (aby ex) Simplify these expressions: . a xxxt bIPxar € g @ 6x5 3x7 € (WP x27 abe This is the base, Cans Expand these expressions and simplify if possible: a -3x(7x-4) b y°B-2y4) © 4xGx=2x8 +505) d 2x(Sx+3) SOx +3) PMs Gass) Crate a -3x(7x— 4) = 21x? + 12 b yAS ~ 2p%) = By? - 2y® © AxGx~ 2x2 + 538) = 0x8 ~ Bx? + 20x" d 2x(5x +3) - 5(2x+ 3) = 10x? + Gx 10x- 15 10x? - 4x— 15 | Goo | Simplify these expressions: 3x2= 6x 20x? + 183% Ze 6 Se fl vm 1. Simplify these expressions: a a xat b 2x x 3x? ef ae ee Fo Ip ae g 10x82 28 h (pp spt i QwPs 20 i 8p = 4p k 2a x 30° ae m 9x2 x 302) 3x3 x 2x? x dys 0 Tat x Gat)? Pp Gy £299 4 2a + 3a? x 6a br 3a x 2a xa? ata) SG} 2 Expand and simplify if possible: a 9x=2) b xv+9) © -3y(4-3)) d xiv +5) fe -x3x+5) f -Sx(4x-41) g Gx 45)x bh -3)(5 299) i -2x(5x-4) i Gx-5)? k 3(e+2)4 00-7) 1 5x-6-(3x-2) m (c+ 3d2)—3Q2c+d2) ow (P4324+9)- OP +328-4) 0 xG3x? = 20-45) p 7y°2-5y+39) a 25 = Ty +3) F 1(x-2)+ Hx4 4) -6(x-2) $s Sx- 34-20) +6 t axe aG—4x)+7 wu date + 3)- 200-7) ¥ BM NH1)-SeBe-4) 3 Simplify these fractions: 9 Oxt+ 10x6 2x 8x3 + Sy ano @®) Expanding brackets To find the product of two expressions you multiply each term in one expression by each term in the other expression, Multiplying each of the 2 terms in the first expression by each of the a3 3 terms in the second expression gives 2 x 3 = 6 terms. Bee (0 5Yldx~ by +3) = alr 2y +3) + 5402p +3) = 4x? ~ 2xy + 3+ 20x~ 10y + 15] 5x ax —2xy +23x—10y 415 EMD ED wove Expand these expressions and simplify if possible: a (x +5) +2) b (w= 2+ 1) © 3 d (x+y)Gx=2y-4) ‘Simplify your answer by collecting like terms, 2 (e+ Bet 2) =x + 2x4 5x4 10 Bx24 7x10 aa assy Crate © nF vie = Y) xy ay ty ax aye 4 (+ Bx - 2y- 4) (Sx = 2y ~ 4) + (Sx = 2y~ 4) xe = 2xy— Ax + Sup — 2p? - Ay x2 4 xy ~ Ay ~ By? — ay Expand these expressions and simplify if possible: a xx +3)0-7) b x(x -3)Qx-y +4) © (x AOr + 3+ DD | a x(2x + 3x7) b x(x ~ Syllax- y+ 4) Sx* — Sxyl@x —y +4) = Sx(2x—y + 4) ~ Sxy2x © iw Act Bx + x2 = x= 12)ee +) EID mn 1 Expand and simplify if possible: a (et ArH 7) b (x= 3042) © (v-2P d (x—y)Q2x +3) © (x + 3ydx—y) £ Qv—4y8x+y) 2 2x 30-4) h Gy42yP i Qx+8)2x+3) J (e+ 5)Qx 4 3y-5) k (= DGx-4y-5) 1 4) t +5) m (+2 1+ 3) n Qx42p+ 3046) (4-yyy-x +3) P (Ay + SW3x- y +2) @ (Sy = 2x + 3x4) (Gy -x = 25 -y) ata) SG} 2 Expand and simplify if possible: a Stet Ir-4) x= Dx +5) © 3(x=3)(r-3) @ x(x ye +y) xQx + 33x44) £ y(e-Syr4 1) 8 Bx 2y)4x +2) HT = x)Q2x- 5) i xQx+y)Sx-2) 1 yGx+2y-3Qe+)) m xQx4 3x45) 2xBx-dx-y—3) 0 3x(x-2IQe+3y+5) P r+ 34 04D) (xt WOAH HF LV= DOr= 5) S$ (x= Sr AYxr-3) xt Noe= ees) w Ax+9Bx-1e+2) b e h j x(v + 2)(e + 3y-4) k prt y= Der+5) n a t ¥ Gx-2Qx+)Gx-2) wx+y-e-1I) x r= 39h © 3 The diagram shows a rectangle with a square cut out. ET EWES ‘The rectangle has length 3x — y+ 4 and width «+7. ‘The square has side length v2 Use the same strategy as you would use Find an expanded and simplified expression pee ca for the area shaded green in loom xa7 Tem Benya © 4 A cuboid has dimensions (x + 2)em, (2x - 1) em and (2x + 3)em, Show that the volume of the cuboid is (4x* + 12x? + 5x—6)em’ ©) 5 Given that 2x + 5y}Gx~ 2x +y) = ax" + bxty + exp? + dy, where a, 6, ¢and dare constants, find the values of a, b, ¢ and d. (2 marks) Expand and simplify (x+y ® Factorising You can write expressions as a product of their factors. ' Factorising is the opposite of expanding brackets. PMs Gass) Crate EEO ©) ED wuss Factorise these expressions completely: a3ve9 b x= Sy © 8x? + 20x 9x2y-4 15xy?@ 3x? Ixy a Bv49= 3043) b x? 5x = x(¥-5) | © 8x? + 20: 4x(2x +5) | 4 9x2y + 15xy? = Sxy(Su + 54) © 3x" Oxy = 3xtx ~ 35) | # A quadratic expression has the form (CCT eat numbers are onthe postive and ave + bx + ewhere a, band care real ‘negative numbers, or zero, including fractions numbers and a+ 0. oeeren To factorise a quadratic expression: + Find two factors of aethat add up to b———— For the expression 2x°+ 5x = 3, a axe + Rewrite the btermasasumofthese =. 92 v4 6x3 ‘two factors + Factorise each pair of terms + Take out the common factor: near yee-y) CEEEED an exoressionin the form x? — 9? is called the difference of two squares. =x(2x~ 1) +3@x-1) = x= 1x43) am Factorise: -5x-6 b 4 6r48 © 6x2-1xe- 10d 8-25 42-9? x-6 (x +N) lx +. x+ thx = 6) x ex-Gx-G ata) Mesias sy be +exe8 Bxt42x44x48 x(x 2) + A(x + 2) = (x4 2x +4) © Gx?= 1x10 = Gx? 15x 4 4x ~10 = 3x(2x — 5) + 22x - 5) ———+ = (2x- 5)18x +2) 4 xe -25 2g (+ 5x - 5) — | © 4x2 - 992 - 338 (2x + 3y(2x - 33) Factorise completely: aed? b= 25x a 2x? b x3 - 258 ae 2) (x? — 5) (x + S)or= 5) © x94 3x2 — 10x = xix + SXx- 2) 1. Factorise these expressions completely: a 4v48 b 6x24 © 20x-415 © 4x +20 f 6-18 h 2 +4x i oa-x k 10° -5y 1 35x°- 28x n3y42y 0 4x4 12e p 5y°—20y 4 Oxy? + 1204 bab ~ ab? 8 Sx? =25xy t iaxty + 8x wu 1sy = 2092" y 12x30 woye—aey x 12y2— 4px Mesa as) Crate 2 Factorise: a ear b ce 4liy+ md d easel e f e-8r412 gv t5x+6 b i e-3x-10 j vtx-20 k 1 3x°+10r-8 : EMD For paren, sake 2 ouras.a common Boat Maxi 24 factor first. For part p, let y'= qvns rea) s 4x2-25 t 9x2 = 2592 u 36x? 4 y 2x7-50 Ww 6x? 10x44 x 15x74 420-9 3 Factorise completely: a 42x b +x € x8-5x do -9y ea 12 1x84 112 + 30x g 8-74 6x hos 64x i 2x) - 5x? 3x j 2x) 13x? + 15S k 8-4 1 3x3 + 27x? + 60x Qmarks) GEES Watch out for terms that can be written as a function ofa function for example oa (0 @® 4 Factorise completely x4 - y © 5 Factorise completely 63° +7?~ 5x. (2 marks) rs Write 4x — 13x? + 9 as the product of four linear factors. ©) Negative and fractional indices Indices can be negative numbers or fractions. COED rationar humbers are those that can be written as “where b and bare integers, and x0, nterms ' You can use the laws of indices with any rational power. CEEDD oie aistne positive square root ofa. Sara Gay ata) Simplily: bxxx ed 2xt8e4res —e VD5x% V125x¢ = (125x5) (25) CEN 10) TED semenernon Evaluate: ao b 64 © 492 d 2st aa assy Crate Given that y= 6 bay .+2, express each of the following in the form kx", where k and 1 are constants. ED) FETED evoarcu-soume 1 Simplify j Went boxset e wy h Sxiext k Gx) xGky 3 Simplify: a (64x"0) © 4 a Find the value of 814, b Simplify x(x © 5 Given that y ay? 1 by Geen Check that your answers are in the correct form. IF cand m are constants they could be positive or negative, and they could be integers, fractions or surds. xtaw Sc e-0s i 3xtx 2x ee e 87, express each of the following in the form kx", where k and n are constants. ' weds? © (125x")1 at 9x2= 15x5 4 Sete Boe 15x? (I mark) (2 marks) (2 marks) (2 marks) " Pee aise \LGEBRAIC EXPRESSIONS © ss san integer that is not a square number, then any multiple of vit pte of surds are v2, /19 and 5:2. Surds are examples of irrational numbers. CED rational numbers cannot be written The decimal expansion of a surd is never-ending in tne for 2 where a and b are integers. and never repeats, for example v2 = 1.414213562... h called a surd. ‘You can use surds to write exact answers to calculations. ® You can manipulate surds using these rules: Simplify © Sv6 - 2/24 + \294 = WG(5 ~ 20 + 549) = -2x24+7) = 66) bre aa assy Crate OM 23) SLID onrzn-soume Expand and simplify if possible: a (5-03) be Do not use your calculator for this exercise. 1 Simplify: bi © 0 ~ aE da s32, e 90 f oh 22 bh +80 i 30048 § 17S +63 + 2928 k 28-263 +07 1 80-220 + 345 a 1m 3:80 — 220 + 5v45 0 12 + 3V48 + V75 ar 2 Expand and simplify if possible: a /3(2+V3) b 33-3) © V2(4-V3) d (2-923 +05) © (2-V3N3-07) £ + V5)2 +05) g (5-3) -V3) bh (4+ V3)(2-V3) i @-vIp@+vT) © 3 Simplify /75 - /12 giving your answer in the form av’3, where a is an integer. (2 marks) €B Rationalising denominators If a fraction has a surd in the denominator, it is sometimes useful to rearrange it so that the denominator is a rational number. This is called rationalising the denominator. = The rules to rationalise denominators are: + For fractions in the form =, multiply the numerator and denominator by va. + For fractis in the form multiply the numerator and denominator by (a - vB). ae + For fractions in the form _, multiply the numerator and denominator by |a + vb). eat) Mee as Rationalise the denominator: i . B+ a 3 342 S-2 aaa B4v2 (3 + v2 - v2) “9-3/2 + 3/2 -2 3-8 WS + V2) 5 +52 + 25 +2 5-2 | 178-845 4-28 1x (4 + 208) 203)4 + 23) | aa assy Crate oc Do not use your calculator for this exercise. 1 Simplity 1 at S 1 Weir ae a2 3 Rationalise the denominators and simplify: Ons 2 (5 +9/2)3-V2) V3)2+V3) ® 4 Simplity a? 7 siving your answer in the Gare 5 5 You can check that your answer is in the correct form p+ qi5, where p and g are rational camben (marks) fOrmby writing down the values ofp and y and checking that they are rational numbers. Chapter review @) ExEcuTVe FNCTON 1 Simplify: ayy? b 3xtx2xF © xt) + 2x8 4B x 36x bE 2. Expand and simplify if possible: a (v+3w-5) b Qx- 73x41) © Qx+5\Gx- +2) 3. Expand and simplify if possible: a xir+ r= 1) bet r= 347) © 2x +3)=2GX=1) 4 Expand the brackets: a 3544) b Se G—Sy+2x) © SxQv43)—2x(1-3x) od 3x + 3x)- 2B -2) 15 earl Bee 5. Factorise these expressions completely: a 3x2 +x b 432+ 10y ¢ Sa xyexy 4 8x2 + 10x¢y 6 Factorise a 43x42 b 3x2 + 6 ¢ x= 2x-35 © 58-1By-6 Ff 6-Sy=¥ 7 Factorise a 2x3 + 6x b x3 36x € 2x84 7x2 15x 8 Simplify: a 9x3 + 3x3 bay © 9 Evaluate, without using your calculator: 8y 225)! + (5) » Gao) 10 Simplify, without using your calculator: a see b 120 +245 - 80 ves 11 a Find the value of 35x? + 2x ~ 48 when x= 25. b By factorising the expression, show that your answer to part a can be written as the product of two prime factors. 12. Expand and simplify if possible, without using your calculator: a 23+ 5) b (2-v5\(5+03) © (6-V2\"(4-\7) 13 Rationalise the denominator and simplify: acs b— oo. ae i 3 O-1 B-2 B47 +3 7p 14. Do not use your calculator for this question. © 15 @&) 6 @® © @® vo a. Given that x! — x°— 17x —15 work out the values of b and c. x +3)(32 + Ax + 0), where b and c are constants, b Hence, fully factorise x9 x?- 17x ~ 15. Given that y = @jx", express each of the following in the form kx", where k and n are constants. ay! (mark) bay (mark) Show that ——>——can be written in the form va + vB, where a and bare integers. (5 marks) Expand and simplify (VT ~ 5)(5 - v1), without using your calculator. (2 marks) Factorise completely x - 643°, (marks) Express 27°* in the form 3", stating y in terms of » (2 marks) Mesa as) Crate 20 Solve the equation 8 + TT a Give your answer in the form avB, where a and b are integers. (Amarks) ©® 21 Do not use your calculator for this question. A rectangle has a length of (1 + /3)em and area of vI2 em? Calculate the width of the rectangle in em. Express your answer in the form a+ 6/3, where a and b are integers to be found. ¢ : © 22 Show that ‘can be written as 4x7! = 44° (2 marks) ve 23 Given that 243V Given that “can be writen inthe form 4x" +3, write down the value of a and the value of 5. (2 marks) a. Simplify (a + vB)(da — 8). 1 1 b He how that ————+ — a AMIS TERD TEED RS ERS 3 «find the value of a. (GB marks) Sood ts 1. You can use the laws of indices to simplify powers of the same base, sata + (esa Factorising is the opposite of expanding brackets. ae + ansaid" A quadratic expression has the form ax? + bx + c where a, b and c are real numbers and a #0. @+G-y) wnun ‘You can use the laws of indices with any rational power 6 You can manipulate surds using these rules: + ab =Vax yb . 7 The rules to rationalise Seo are: + For fractions in the form 7, multiply the numerator and denominator by Va + For fractions in the form Fe multiply the numerator and denominator by (a - vb). ati + For fractions in the form —1,, multiply the numerator and denominator by (a+b). a-b : a Learning objectives After completing this chapter you should be able to: © Solve quadratic equations using factorisation, the quadratic formula ‘and completing the square Read and use f(x) notation when > pages 19-26 working with functions Sketch the graph and find the turning > pages 25-27 point of a quadratic function “> pages 27-30 Find and interpret the discriminant of a quadratic expression > pages 30-32 Smarr ead Solve the following equations: a 3r+6 b 5(v+3)=6(2x-1) © 4x2= 100 d (x-8)2= 66 ‘# International GCSE Mathematics Factorise the following expressions: a teat 15 b x243x-10 d x2—400 « Section1.3 jphs of the following equations, points where each graph cro: ; by ¢ x42p=18 dy=x € International GCSE Mathematics Solve the following inequalities: axt8<1l b2v-5 © 4x-7<2x-1) d4-x<11 ‘ International GCSE Mathemati ADRATICS 13 or oF) Quadratic functions are used to model tile motion. Whenever an object thrown or launched, its path will mately follow the shape of a CW tes raid ) solving quadratic equations A quadratic equation can be written in the form ax? + bx +.¢=0, where a, b and c are real constants, and « # 0. Quadratic equations can have one, two, or no real solutions. = To solve a quadratic equation by factorising: CED the solutions to an + Write the equation in the form ax? + bx+e=0 ‘equation are sometimes called + Factorise the left-hand side Sa ee ee + Set each factor equal to zero and solve to find the value(s) of x SI) ED msm aie Solve the following equations: a xt-2-15=0 bao © 6x?+13x-S=0 0 d x= Sx+18=2+3x TE This statement says fx +3 =0, then x =—3! a x8 2x-15=0 (x + 3e-5)=0 Then either x +3=0> x or ¥-5=05¥=5 3 Sox =-3 and x of the equation | Qa are the two solutions The solutions are © Gx®4138x-5=0 (x - 2x + 5) = 0 Then either 3x — 1 =O > x or ax+5 The solutions are 4 x8 5x4 1822 43x x - Bx +16 (x= 4-4) When a quadratic equation has exactly one rct is calle a repeated rot You can also say thatthe equation has two equal roots. me aiid CUE} In some cases it may be more straightforward to solve a quadratic equation without factorising. Example [2) Solve the following equations: Senet flict Ia ‘statements in one line of working, You say ‘plus or minus’ or ax=3- B27 The solutions are x = 3+ v7 and 3-7 | sous 1 Solve the following equations using factorisation: attirt2=0 bxttSye4=0 © t47r+1050 9 dP -x-6=0 ee 8xt15=0 9 f -9x42050 — g x°-Sx-6=0 he -4y-12=0 2 Solve the following equations using factorisation a sted bx © Bxt= 6x d Sx? = 30x exe Iet3=0 f 6x¢-7y-3=0 — g 6x?-Sv-6=0 hh dx? 16x + 15=0 3 Solve the following equations: a 3x25. b Qx-3) ¢ (= 7F e3ts5 f (e-3F2 13 g Gr-1P=tl h 5x? 10x? = ~7-4 x42 i 62-72 1x j 4x24 x= 6x- 2 © 4 This shapehasan area of 4am? Divide the shape into two sections: Find the value of x. Joc +3)m i ©® 5 Solve the equation 5x +3 = v3x+7. CW tes Cr aieied 2 ‘Some equations cannot be easily factorised. You can also solve quadratic equations using the quadratic formula. = The solutions of the equation cer ax? + bx + c= O are given by the formula: =b + P= bac 2a CTD treet o2a0 treconsms EE) EXD ween Aer Solve 3x? — 7x -1 = O by using the quadratic formula. Or x = 2.47 (3 si) or rin 1 Solve the following equations using the quadratic formula, Give your answers exactly, leaving them in surd form where necessary. 0135 (3 sf) aetaet b xe 3x-2 ex s6r46=0 d x-Sx-2=0 e 3x+10x-2=0 0 f 4x°—4x- g dxt-Ir=2 h IL? +2x-7=0 2 Solve the following equations using the quadratic formula. Give your answers to three significant figures, a etdes2 be-8rt1=0 e+ IIy-9=0 0 d xe-7e-17=0 e 5x4 9x-1=0 — f 2v-3x-18=0 gg 3x74 8 = 16x bode + lly = 5x18 3 For each of the equations below, choose a suitable method and find all of the solutions. Where necessary, give your answers to three significant figures. a e+8yt12=0 — b x249x- 1150 © P-9x- 150 d 24 5x42=0 MLD You can use any method ‘you are confident with to solve these equations. © Qt 8F=100 Ff 6x2+6= 120 g = Mee hvs Be 15 Pea aid ©® 4 This trapezium has an area of 50m? Show that the height of the trapezium is equal to 5(\5 ~ 1) m. tues Height must be positive. You will have to discard the negative solution of your quadratic equation. —erlm—> Challenge EBD rte te equation nthe form - r 7 ‘ax? + hx + ¢ = Obefore using the quadratic Given that xis positive, solve the equation aptsae carer i x x42" 195) ©) completing the square It is frequently useful to rewrite quadratic expressions by completing the square: . (xa) (27 wie=(x+3) -(3) You can draw a diagram of when xand bare positive: process The original rectangle has been rearranged intothe © <——x—>=0> shape of a square with a smaller square mi ‘The two areas shaded blue are the same. at ebx AGM ESMMNHEE Complete the square for the expressions: ee ee a4 8x b t= 3x © 2-12 a x4 Bx= (vt 4? - 42 = (x44) = 16 © 2x? — 12x= 2[x? - Gx) = 2iix = 3 = 2x = 3° gata ait = 2ly - 3" - CW tes raid Write 312 + 6x + 1 in the form p(x + g)? +r, where p, q and rare integers to be found. a eeret (rns Bi? + 2x) +1 (ix + Pe Bix + F-3 +1 (x Pe Sop=3,q=1andr=-2 erry €3) INTERPRETATION Gz) E> © ircvesons 1 Complete the square for these expressions: ‘write the expression as 2 2 3 wax? — 16x + 10 then avtedy bxt-6x e -l6y dex eH Ide ee ror wont of the first two terms to get alr? + 4x) +10. 2. Complete the square for these expressions: a 2x24 6x b 3x2 24x © Sx°+20x d 2x?— Se @ Bx— 2x 3. Write each of these expressions in the form p(x + g)? + r, where p, g and r are constants to be found: ade48rtl 5 © 4 Given that x? + 3x +6 © § Write 2 + 0.8% ~ 0.04x? in the form A ~ Box + CP, where A, Band Care constants to be determined marks) was Solve the equation x + 8x + I Give your answers in surd form. 15x43 © 32+2v-1 d 10-16y—4x? © 2y-8x2 +10 +a)? +6, find the values of the constants a and b. (2 marks) 0 by completing the square, x + 6x+10 me aiid 5 EI Cy ax Solve the equation 2x? — 8x + 7 = 0. Give your answers in surd form. EEcuen This is an equation so you can divide every term by the same constant. Divide by 2 to get x? on its ‘own, The right-hand side is 0 so itis unchanged. 2x? - Bx +7 stax} So the roots are 24 Sandx=2- EDD ec jeratatmnrindea bs ‘Solutions to quadratic equations quickly. ss 1 Solve these quadratic equations by completing the square, Leave your answers in surd form, ave6rel=0 bare 1243 d= 10x © x 44y-2 2. Solve these quadratic equations by completing the square, Leave your answers in surd form, a 2H6x-3=0 — b Sx7#8x-2=0 © 4x7-¥-8=0 @ 15-6x-2x7=0 @© 3 2~ 14 41 = (0 + 9)? + gy where p and g are constants, ‘a Find the values of p and g. (2 marks) b Using your answer to part a, or otherwise, show that the solutions to the equation x? = 14x + 1 =0 can be written in the form r + sv3, where rand s are constants to be found. (2marks) 4 By completing the square, show that the solutions to Problem-solving the equation x? + 2by + ¢ = Oare given by the formula Follow the same steps as you would xen bab GAmarks) the confilents were umbers. ‘a Show that the solutions to the equation ax? + 2bx + e= 0 are gven by x=—5 2 P=ae GED stare oy dividing the whole ay a ‘equation by a. b Hence, or otherwise, show that the solutions to the ‘equation ax? + bx + ¢= 0 can be written as You can use this method to prove the quadratic formula, + Section 2.1 CW tes raid @&) Functions ‘A function is a mathematical relationship that maps each value of a set of inputs toa single output. The notation f(x) is used to represent a function of x. = The set of possible inputs for a function is called the domain, = The set of possible outputs of a function is called the range. This diagram shows how the function f(1) five values in its domain to values x? maps range. © The roots of a function are the values of x for which f(x) = 0. ‘The fonotonsF and g are given by fl) =2— 10 CEE the input oro urction, and g(x) =? - 9, ER, “x, can be any real number, then the a Find the values of £(5) and g(10). domain can be written as x € R. b Find the value of x for which f(x) = g(x). ‘The symbol € means ‘is a member ‘a {(5) = 25) - 10 =10=0 set of real numbers. g(l0) = (OP - 9 = 100-9 =91 b fix) = g(x) 2x-10=x7-9 x=1 ! Ge The function f is defined as fx) = x2 + 6 a Write f(x) in the form (x + p+ 4. b Hence, or otherwise, find the roots of f(x), leaving your answers in surd form. © Write down the minimum value of f(x), and state the value of 2 for which it occurs, -5,xER. afi)=x@ter-5 —] = (r+ 3-9-5 xe 3e-t4__| wt aViA -34 4 f(x) has two roots: 3 + Vid and -3 - id 25 aie} L So the minimum value of f(x} Find the roots of the function fx) = x4 + 7x3 = 8, «ER, io x 47x58 = 0) (2)! + 703) - B= 0 (3-169 +6) The roots of (x) are 1 and ~2 \_|_[oetonceearana | (4) can be written as a function of a function. The only powers of x in f(x) are 6,3 and 0 so you can write it as a quadratic function of » Altematively, let u fia) = a8 + 78 = 8 = 0x8)? + 7h" 6 an > ____ =) = thee 8) So when fox) = O,u= 1 or w Wustexeetoxet (aang The roots of f(x) are 1 and GEES BD vercve 1 Using the funetions f(x) = 5x+ 3, g(x) = x2 ~ 2 and h(x) = VT, find the values of a fl) b a3) © hg) di f1.5) e 22) a) f hw) £14) +22) bh MO)+g@)+hO) i & solving Substitute x =a into the function and set the resulting expression equal to 8. ® 2 The function f(x) is defined by fx) = x7 - 2x, x ER. Proble Given that f(a) = 8, find two possible values for a. 3. Find all the roots of the following functions: a f(x) = 10 15x b gx) = (0+ 9-2) © h(x) =5?+ 6x40 d iQ) = 144-2 © KOS ae SMH T) mlx) sx 4 Sx? = 24 ©) 8 The function f is defined as fix) = raid 4 The functions p and q are given by pix Find the two values of x for which pt 3x and q(x) = 2x 6.¥ER 5 The functions f and g are given by f(x) = 2x3 + 30x and g(x) = 17x?, x ER. Find the three values of x for which f(x) = g(). 2-2x+2,.xER. © 6 The function f is defined as fx) a Write fix) in the form (x + p)? + g, where p and g are constants to be found. (2 marks) b Hence, or otherwise, explain why I(x) > 0 forall values of x, and find the minimum value of f(x). (i mark) 7 Find all roots of the following functions: a fix) =xo + 9x9 +8 b g(x) x4 = 12x24 32 ED The function in h(x) = 27x + 26x -1 a j(x) = 320! 330841 part bas four roots. -We +10 £ mx <= 28039) +27, ER, Mee) 2st See +2xt-12 e K(x) a Write fix) in the form (3° = a)(3* =D), : where « and bare real constants, marke) ee function ofa function Hence find the two roots of f(x) (2 marks) @ Quadratic graphs When f(x) = ax? + by + ¢, the graph of y= f(x) has a curved shape called a parabola. You can sketch a quadratic graph by identifying key features The coefficient of x2 determines the overall shape of the graph When a is positive, the parabola will have this shape: \/ When a is negative, the parabola will have this shape: /\\ © The graph crosses the y-axis when x=0. The y-coordinate is equal t0 @ The graph crosses the x-axis when y=0. The x-coordinates are rots of the Function fs) © auachatie graphs have one turning paint This can bea minimum or a maximum Since a paraboia is symmetrical the turing Pointand line of symmetry are half-way between the two rots. * You can find the coordinates of the turning point rs cacupeee: of a quadratic graph by completing the square. ED re cap sty neo +ais f(a) = abe + p)? + g, the graph of y= fx) has a yay 2) © omen turning point at (-p, 4). yeaxtby (7 ection Pd Sketch the graph of y =x? Sx +4, and find the coordinates of its turning point Asam 116 posttve, the graph has 3 VJ shape and a mirimum point. Whee x= 0, ¥= 4,20 the gfaph crosses the y-axis at (O, 4). When y = 0, -5x44=0 (= r-4)=0 em lor = 4,20 the graph crosses the Sais at (1,0) and 4, 0) | Completing the square: | ieee = (x3) So the minimum point has coordinates Bux4): Ce Alternatively, the minimum occurs when xis half-way between 1 and 4, 20 the minimum has coordinates xe 5x4 4s (x The sketch of the graph is: Cm CW tes raid Sketch the graph of y= 4x ~ 2x? 3, Find the coordinates of its turning point and write down the equation of its line of symmetry. As am =2 lp negate, the araph has a /\, shape and a maximum point When x = y= -3, so the graph xis at (0, -3) | crosses the y When y = 0, -2x? + 4v-3 = 0. Using the quadratic formula, There are no real solutions, so the graph does not cross the x-axis, Completing the square: ~2xt + Ae 3 Me) te — IP + 2 2x = 1 So the maximum point has coordinates =. The line of symmetry i vertical and goes through the maximum point. It has the ‘equation x = 1 Pe) alae) CUE} GEE) EID ws 1 Sketch the graphs of the following equations. For each graph, show the coordinates of the point(s) where the graph crosses the coordinate axes, and write down the coordinates of the turning point and the equation of the line of symmetry. a ysxr-6x4+8 by 24+ 2x-15 e@ yet 6x47 f ps2 tax +10 d paxt+3r42 h y= 6x2 19x +10 bree. Gncaenr Check your answers. by substituting values into the function. In partethe graph passes through (0, 1), so h(0) should be -18, 2. These sketches are graphs of quadratic functions of the form ax Find the values of a, band c for each function, 3. The graph of y= bx +c has a minimum at (5, ~3) and passes through (4, 0). Find the values of a, 6 and . (Smarks) @) The discriminant Ifyou square any real number, the result is greater than or equal to 0. This means that if y is negative, \y cannot be a real number, Look at the quadratic formula: IF the value under the square root sign is negative, x cannot be {a real number and there are no real solutions. Ifthe value under the square root is equal t0 0, both solutions will be the same * For the quadratic function f(x) = ax? + bx + ¢, the expression b? - 4ac is called the discriminant. The value of the discriminant shows how many roots f(x) has: ‘+ If? ~ 4ac > 0 then f(x) has two distinct real roots. + If? ~ 4ac = 0 then f(x) has one repeated root. + IFO? — 4ac < O then f(x) has no real roots. CW tes raid You can use the discri nant to check the shape of sketch graphs. Below are some graphs of y= f(x), where f(x) = ax? + by +e. SECM 23) ELIE rvoeuensoumc Find the values of k for which fx) = 2 + kx +9 has equal roots. Problem-solving Here a= 1, b=kande=9 Use the condition given in the question to write a For equal roots, 6? ~ 4ac statement about the disc Ke -4x1x9=0 k= 36=0 ke = 36 sok = 36 Find the range of values of k for which x2 + 4x + & = 0 has two distinct real solutions, xe+4x+k=0 Here a=1,b=4 ande=k. For two real solutions, b® - 4ac > 0.1 #-4x1xk>O 16-4k>0 16 > 4k 4>k Sok<4 Erm airy CUE} XEED ED veo 1 a Calculate the value of the discriminant for each of these five functions i fiat + 8x43 fi g(x)= 2-344 fii h(x) =—x? + Tx -3 iv j= - 8x4 16 Vv k(x) =2v— 32-4 b Using your answers to part a, match the same five functions to these sketch graphs. AV AMY Find the values of k for which x2 + 6x + k= 0 has two real solutions. (2 marks) Find the value of ¢ for which 2x ~ 3x + 1= 0 has exactly one solution, (marks) Given that the function f(x) = sx? + 8x +s has equal roots, find the value of the positive constant s. (2 marks) Find the range of values of & for which 3x? 4x + k = 0 has no real solutions. (marks) ‘The function g(x) =.2 + 3px + (4p ~ 3), where p is an integer, has two equal roots. a Find the value of p. (2 marks) b For this value of p, solve the equation x? + 3px + (14p 3) (marks) T h(x) = 2x7 + (k + A)w + k, where k is a real constant, ees a Find the discriminant of h(x) in terms of k, (marks) Ifa question part says hence or b_ Hence or otherwise, prove that h(x) has two distinct otherwise" its usually easier to use your real roots for all values of &. Bmarks) _2nsWer to the previous question part. Prove that, if the values of a and e are given and non-zero, itis always possible to choose avalue of b so that f(x) = ax? + bx +c has distinct real roots. b Isit always possible to choose a value of b so that f(x) has equal roots? Explain your answer, CW tes raid CEST 2) IED cccrrvermenon 10 Solve the following equations without a calculator. Leave your answers in surd form, where necessary, 0 b 3x24 13x-10=0 7 a yeeaye © Sx? = 10y = 4y +3 d (2x-sp Sketch graphs of the following equations: etxn3 = 15x - 2? a yaessyed © y=6-10x~4: e f(x) =8 + 3y = Sand g(x) = 44+ &, where k is a constant, a Given that £(3) = g(3), find the value of k. (Gmarks) b Find the values of «for which f(x) = g(x). (Gmarks) Solve the following equations, giving your answers correct to 3 significant figures: a +Mk-1=0 b 2e-Sr+1=0 © 10 d Bx Write each of these expressions in the form p(x + 4)? + r, where p, q and r are constants to be found: a+ 12v-9 b Sx? 40+ 13 © 8y— 25? d 3x2- (041)? Find the value k for which the equation 5x* ~ 2x + &=0 has exactly one solution, (2 marks) Given that for all values of x: 3x2 + xt Se ple gh tr a find the values of p, q and r (3 marks) b Hence solve the equation 3x2 + 12x + 5 =0. (marks) The function f is defined as f(x! 20(2") + 64,8 ER. a Write f(x) in the form (2 ~ a)(2* ~ 5), where a and b are real constants (2marks) b Hence find the two roots of f(x). (2 marks) Find, as surds, the roots of the equation 2(x + I= 4) = (r= 27 =0. Use algebra to solve (x - 1)(x +2) = 18. Sal sie CUE} 11 A diver launches herself off a springboard. The height of the diver, in metres, above the pool f seconds after launch can be modelled by the following function: h(n) = Sr—10P + 10,1 =0 a How high is the springboard above the water? (1 mark) b Use the model to find the time at which the diver hits the water. (3marks) © Rearrange h(i) into the form A ~ B(¢ - CP and give the values, of the constants 4, Band C. (3 marks) d_ Using your answer to part ¢, or otherwise, find the maximum height of the diver, and the time at which this maximum height is reached. (2 marks) 12. For this question, f(x) = kx? + (4k + 2) + 1, where k is real constant. a Find the discriminant of f(x) in terms of &. GB marks) By simplifying your answer to part a, or otherwise, prove that f(x) has two distinet real roots for all non-zero values of & (2 marks) ¢ Explain why f(x) cannot have two distinct real roots when k= 0. (mark) 13 Find all of the roots of the function r(x) = x* = 17x + 16. (S marks) a The ratio of the lengths ad in this line is the same as the ratio of the lengths bc. —s Show that this ratio is 2°? 1 b Show also that the infinite square root 145 Vrey14yi+vte vs CW tes ode 1 raid 1 To solve a quadratic equation by factorising: + Write the equation in the form ax? + bx +¢=0 + Factorise the left-hand side + Set each factor equal to zero and solve to find the value(s) of x. The solutions of the equation ax? + bx + ¢ = Owhere a + 0 are given by the formula: ab = \bE= hae xerbre (x44) (4) attbrtcma(s+2)'+(c-2) The set of possible inputs of a function is called the domain. ‘The set of possible outputs of a function is called the range ‘The roots of a function are the values of x for which f(x) = You can find the coordinates of a turning point of a quadratic graph by completing the square. If f(x) = a(x + p)? +g, the graph of y = f(x) has a turning point at (-p, q). For the quadratic fun n f(x) = ax? + bx +e = 0, the expression b? — 4ac is alled the discriminant. The value of the discriminant shows how many roots f(x) has: + If 6? —4ae > 0 then the quadratic function has two distinct real roots. + If B® — 4ac =0 then the quadratic function has one repeated real root. + If B® —4ac <0 then the quadratic function has no real roots. 3 EQUATIONS rN en Vana Sse Pu eed After completing this chapter you should be able to: Solve multaneous equations using elimination or substitution “> pages 37-38 Solve simultaneous equations: one linear and one quadratic “> pages 39-40 Interpret algebraic solutions of equations graphically > pages 40-43, Solve linear inequal > pages 44-46 Solve quadratic inequaliti Interpret inequalities graphically ar and quadratic inequalities g ~ pages 51: De guerie ees B= Write di numbers in of these sets: a An’ b (4UB) + International GCSE Mathematics of intersection with the axes and the coordinates of the turning point b y=(x-2'+4 \d scientists use regions on graphs to optimise athlet ‘nutritional intake and ensure the} the minimum dietary requirements for calories and itamins, USE UE SU Vals} COGaitoky Ed Linear simultaneous equations in two unknowns have one set of values that will make a pair of equations true at the same time. The solution to this pair of simultaneous equations is x= 5, xe3ysl1 (I) 5+3(2)=5+6=11¥ 4x-5y=10 2) 45) ~ 5(@) = 20-10 = 107 Linear simultaneous equations can be solved using elimination or substitution. EEN) ED wou nine Solve the simultaneous equations: b dx Sp 6x +2y olution is. x = 34, y= 2 EC aie} ee SEWED EVs) Solve the simultaneous equations: he solution is x rons 1 Solve these simultaneous equations by elimination: a 2v-y=6 © Sx42y=6 4x +3y 3-109 = 26 d2x-y f 3x48 6x4 2y 2 Solve these simultaneous equations by substitution: x43 b 4x—3y=40 © 3x-y=7 a etys 10x + 3p =-2 3 Solve these simultaneous equations LD Fist scavenge a 3x-2y +520 > © 3y=5(x-2) both equations into 3 the same form, Sirty)=Ox+1) 2x +3y44=0 3w-D+y+4=0 egaxtbyae 4 Bvteky=8 are simultaneous equations where & is a constant. ‘constant, as the a Show that x= 3. (3 marks) same value in both equations. b Given that y=}, determine the value of k. (1 mark) 5 2x py axe Sytq= are simultaneous equations where p and gare constants. ‘The solution to this pair of simultaneous equations is x = g, y= —1. Find the value of p and the value of g. (Smarks) USE UE SU Vals} COGaitoky 39 You must be able to solve simultaneous equations where one equation is linear and one is quadratic. To solve simultaneous equations involving one linear equation and one quadratic equation, you need to use a substitution method from the linear equation into the quadratic equation. = Simultaneous equations with one linear and one quadratic equation can have up to two pairs, of solutions. You need to make sure the solutions are paired correctly. The solutions to this pair of simultaneous equations are x= 4, y=—3 and x yet Q) yebaye2ee5 | (Q) av and 55 LF NS) +20) = 9-12 B= SV and Gx») CAS? + (65)C15) +2155) =225-825 + 11-57 oa | 9 - lay +4? 4 Oy GF 2y2 + 3p+1=0 y+ My +920 oxedorx= Solutions are and x =5,y EE 28) FLIED voeron scume 1 Solve the simultaneous equations axty=ll b2xty=l © y=3x 29 = 30 eeeel d 3a4+b=8 e Quty=7 3a? + b= 28 w=6 2. Solve the simultaneous equations: a Qv42y=7 bv x 4y © Sy-4y Say +2y2=0 Cali) ee SEWED EVs) 3 Solve the simultaneous equations, giving your answers in their simplest surd form: (CotRaRERIEE ‘substituting an expression into an equation. ax-y=6 aye4 (6 marks) show that x? = 2x—11 =0. (2 marks) b Hence, or otherwise, solve the simultaneous equations giving your answers in the form a v3, where a and b are integers. (S marks) © 6 One pair of solutions for the simultaneous equations SIS ne yake-5 4x2 — xy =6 is (1, p) where & and p are constants. a Find the values of k and p. b Find the second pair of solutions for the simultaneous equations. Challenge = wae Given that the simultaneous equations have exactly one pair of solutions, show that kaa IF Qp) isa solution, then x= 1, =p satisfies both equations. ©) Simuttancous equations on graphs You can represent the solutions of simultaneous equations graphically. As every point on a line or curve satisfies the equation of that line or curve, the points of intersection of two lines or curves satisfy both equations simultaneously. «= Solutions to a pair of simultaneous equations represent the p< GEE) BED wore 8 On the same axes, draw the graphs of: 2v43y=8 3 its of intersection of their graphs. Br-y b Use your graphs to write down the solutions to the simultaneous equations. USE UE SU Vals} ai Find the point of Tees calor using technology. b The solution is (7, -2) or x = 7 y a On the same axes, draw the graphs of: Qtys3 yex-3vel b Use your graphs to write down the solutions to the simultaneous equations, a 1 ] on | l | 5 p49 7 $4 fe fai) tT? x+ye3 b The solutions are (+1, 5) or x SN eee and (2, -1) oF x points of intersection. The graph of a linear equation and the graph of a quadratic equation can either: * intersect twice itersect once + not intersect ‘After substituting, you can use the discriminant of the resulting quadratic equation to determine the number of points of intersection. a OSU us} Ceara EQUi ' For a pair of simultaneous equations that produce a quadratic equation of the form ax? + bx +0=0: + Bhac>0 + Bahac=0 + P4ac<0 ‘two real solutions one real solution no real solutions wm VA UN ‘The line with equation y = 2x + | meets the curve with equation kx? + 2y + (k~ 2) =0 at exactly ‘one point. Given that k is a positive constant ‘a find the value of & | Explore how the value of OB b for this value of k, find the coordinates of sthe line and the curve the point of intersection. [te etc ax+1 tt) kat + By + k= 2)= kee + 20x + D+ k= by? bye 24k kat tae Geter ama You are told that the ine meets the curve at exactly one point, so use the discriminant of the resulting quadratic. There wil be exactly one solution, 50.5? —4ae =0. b 2x2 +44 220 ye 2a Point of intersection is (-1, 1), BT UNSEEN SU aus) COGaitoky EEE) ED rover 1 Ineach case: i draw the graphs for each pair of equations on the same axes find the coordinates of the point of intersection. Be- b y=2v-7 © ye3-x ye8-3x +2 xty+1=0 y 2 a Use graph paper to accurately draw the graphs of 2y-= 2x + 11 and 3x5 on the same axes b Use your graphs to find the coordinates of the points of intersection. © Verily your solutions by substitution. 3a On the same axes, sketch the curve with equation x? + y= 9 and the line with equation 2x + y= 6. b Find the coordinates of the points of intersection. ¢ Verify your solutions by substitution. 4a On the same axes, sketch the curve with equation ; , GED sou need to use algebra (v—2) and the line with equation y = 3x -2. nese tose e Be ea b Find the coordinates of the point of intersection. ppart b to find the coordinates. 6 Find the coordinates of the points at which the line with equation y = 3x — 1 intersects the curve with equation y? = xy +15. ©® 7 Determine the number of points of intersection for these pairs of simultaneous equations. 6x? +3x-7 b yay? 18+ 40 22ved yedxt8 y= l0x-9 G@) 8 Given the simultaneous equations Dey x + dky + 5k=0 where k is a non-zero constant a show that x2 + 8kx +k =0. (2 marks) Given that x2 + 8k + & = 0 has equal roots b find the value of (Bmarks) © for this value of &, find the solution of the simultaneous equations, (3 marks) rc Ce arse} ee SEWED EVs) You can solve linear inequalities using similar methods to those for solving linear equations. = The solution of an inequality is the set of all real numbers x that make the inequality true. NTI) ED see Find the set of values of x for which: a Syt9= 0420 b12=3x< 27 CREED you can write the solution to this © ¥x-5)> 5 2Ax-8) inequality using set notation as (x:x = 2.75). This means the set of al values «for which xis pene greater than or equal to 275, a 5x49 Ax+9= 20 b 12-3y<27 3x < 15 © 3(x-5)>5-2(r-8) Bx-15>5- +16 Syo5 416415 Sx >36 x>72 You may sometimes need to find the set of COED ih set notation values for which two inequalities are true Sar eae eT together. Number lines can be useful to find peanarretten eee eta) the solution, SL orx>3is written (x:x< =U (x: ¥>3} For example, in the number line below the solution set is.x > —2and.x = 4, © is used for and means the end value is not included, |__. eisused for and = and means the end value is included. ‘These are the only real values that satisfy both equalities simultaneously, so the solution is -2 3. SoS Here there is no overlap and the two inequalities have = % os to be written separately as x= —1or.x>3. USE UE SU Vals} ai emo Find the set of values of x for which: a 3y—Sx-8 b x-S>1-xor 1S-3x>5 42x a 3r-5x-8 2ax-5<8 4x>-8 2x<13 x>-2 x<65 So the required set of valves 18-2 t-x 15 -3x>5 42x ax-5>1 10- ax > 2x 2x>6 10> 5x x>3 2>x x2 4 2 0 2 4 € 6 o——> «3 The solution is x> 3 or x < 2, BETTE ies 1 Find the set of values of x for which: a 2v-3<5 b Sy+4=39 © 6x-3>2v47 d Sx+6<-12-x © 1Saxe4 f 21-2v>843x g l+x<2543x bh Ix-7<7-7x i 5-05e>1 j Sx+4> 12-26 ri) Cali) ee SEWED EVs) 2. Find the set of values of x for wh a %x-3)>0 b &(1-x)>x-1 © 3x47) <8-x% d %xe-3)-(e+1N<0 — e 141-9 <10(x-4) f Yx-5)=34-x) g 1tv-3(x- 3) <45 he x-2%5+2y< 11 i x(x-4)= 842 | 2 > « - dxOe +3) jxG-ye34x-x8 k 3x4 2x(v=3) S$ 5 #4) 1 aQv-5)< 9 3. Use set notation to describe the set of values of x for which: a v= 2)> x -4and 4v+ 12> 20417 Dv -S 23-x 2x 3 >2and Hx +2) <12+x 15 =x < 2(11 = x) and 5(3x = 1) > 12x 419 3x +8 < 20 and 23x-7) = x+6 5x43 <9 or 52x + 1)>27 1- 6x 43x +7) 5 20 of 23x = 5) = A=(x:3x+5>2) 2 C=(:Me<2x-1) Given that 4.9 (BUC) = fv:p-< x= 4} Ufvix> A find the values ofp, and r @® Quadratic inequalities * To solve a quadratic inequality + Rearrange so that the right-hand side of the inequality is 0 + Solve the corresponding quadratic equation to find the critical values + Sketch the graph of the quadratic function * Use your sketch to find the required set of values. ‘The solutions to the quadratic inequality wae 5 x*=4x~5>Oare the xvalues when = G+ De-5) the curve is above the x-axis (the darker , part ofthe curve) This is when x-<=1 oF {x> 5.In set notation, the solution is (er<-I}Ute:x> 5) The sketch shows the graph of f(x) The solutions to ffx) = 0 ‘The solutions to the quadratic inequality pre-e Lend = 3 [| x= 4-5 -Landx <5or-1d 3 ~ $x — 2x < 0 (y'<0) for the outer parts of the ‘graph, below the x-axis, as shown by the lighter parts of the curve. Inset notation this can be written as feix< Ube x> 2 a Find the set of values of x for which 12+ 4x > 3°. b_ Hence find the set of values for which 12 + 4y > x? and 5y~3>2. a 1244x> x2 You can use a table to check your solution. O>x = 4x—12 ~2exeé x? 4y-12<0 Use the critical values to split the real number x? - 4x ~12=0 line into sets. (w+ 2X -6)=0 xe-2orxe6 Sketch y xe a 5 ey i x-6 = E ie (eea(e-6 | + = + For each set, check whether the set of values makes the value of the bracket positive or negative, For example, fx <-2, (x +2) is negative, (x 6) isnegatlve, and (x + 2\(x-6) Is win ax -12<0 (neg) x (neg) = positive. Solution: -2 x? gues -2 2 gives x > 1 42624668 Bamed o> «m1 The two sets of values tex<é, Sot Find the set of values for which £> 2, x #0 1p where olution is 1 < x < 6. fae Gx > 2x? Gx- 2x20 The solution is 0 1 b 12-x-a?>0 @ 74 13x-2x2>0 h -247x-3x2<0 k 5x30 b l0and 10-2x<5 © 4x2 = 3x1 0and v4 17>2 f a?-2y-3 0. (4 marks) © 7 Find the set of values of x for which a 23x-1)<4-3y (2 marks) b 2x?-5y-3<0 (4marks) € both (31) <4— 3x and 2x2 Sx -3< 0 (marks) @® 8 Given that #3, find the set of values for which 5 <2. Ree x Multiply both sides of the (marks) inequality by (x37. EP) 9 The equation kx? - 2kx + 3 = 0, where k is a constant, has no real roots. Prove that k satisfies the inequality 0 = k-< 3. (4 marks) @ Inequalities on graphs You may be asked to interpret graphically the solutions to inequal of functions that are related to them. ies by considering the graphs = The values of x for which the curve y= f(x) is below the curve y= g(x) satisfy the inequality fa) < gt. = The values of x for which the curve. fa) > a. f(a) is above the curve y= gia) satisfy the inequality Ce arse} ee SEWED EVs) (x) is above g(x) when x-< 2 and when x>5. These values of x satisfy fx) > gla). These values of x satisfy fx) < BU. The solutions to f(x) = glx) are x= 2and.x=5. Ly has equation y Lz has equation y ‘The diagram shows a sketch of L, and L, on the same axes a Find the coordinates of P, and Ps, the points of intersection. Hence write down the solution to the inequality 12 + 4x > x, a x #12 44x =4x-12=0 (= Gir + 2) =6ondx Jostitute into y = x whenx=6,y=36 P, (6,36) 2y=4 P2(-2,4) b 12 + 4x > x2 when the graph of Ly is above the graph of when x ~2ex fla) represents the points on the coordinate grid above the curve y = flx). All the shaded points in this region satisfy the inequality y > f(x) All the unshaded points in this region satisfy the inequality y < f(s) Ca arse = IF y> fle) ory< f(x) then the curve y= f(x) is not included in the region and is represented by a dotted line. = Ify > fa) ory < f(a) then the curve y= fla) is included in the region and is represented by a solid ir IO 13) EES werrrersnon On graph paper, shade the region that satisfies the inequalities: ye -dx<5,y<3x+2and x>0 On graph paper, shade the region that satisfies the inequalities: dyex-2 xe ytr3 4 Ona coordinate grid, shade the region that satisfies the inequalities: yoxt-Dand y<9-a2 5 Ona coordinate grid, shade the region that satisfies the inequalities: yo (x-3P, yt xe Sandy x + 13. Q marks) b Solve the inequality x° - Sy — 14> 0. (marks) © _ 9 Find the set of values of x for whieh (x ~ 1)(x ~ 4) < 2x4), (6 marks) © 10 a Use algebra to solve (x — 1)(x + 2) = 18. (2 marks) b Hence, or otherwise, find the set of values of x for which (x ~ 1)(x+2)> 18. Give your answer in set notation (2 marks) 11 Find the set of values of for which: a 6x-7<2n43 (2 marks) b 2? lly 45<0 (marks) © 5<¥ (4.marks) ¥ @ both 6x —7< 2x4 3and 2x? 1x +5<0. (2 marks) Be ewo (smarts Find the set of values of x that satisfy Find the values of & for which kx? + 8x + 5 = 0 has real roots, (3 marks) The equation 2x? + 4k ~ Sk = 0, where k isa constant, has no real roots. Prove that k satisfies the inequality -} g(x) (Lmark) © 16 Find the set of values of x for which the curve with equation y = 2x? + 3x — 15s below the line with equation y = 8 + 2x (Smarks) © 17 Ona coordinate grid, shade the region that satisfies the inequalities: poste 4x—I2and y<4—¥ (Smarks) 18 a Ona coordinate grid, shade the region that satisfies the inequalities ytxc6y<2v49y>3andx>0 (Gmarks) b Work out the area of the shaded region. (2 marks) Crs 11 Find the possible values of k for the quadratic equation 2kx? + Skx + 5k ~3=0 to have real roots. 2 A straight line has equation y = 2~ kand a parabola has equation y= 3x2 + 2kx + 5 where k is constant. Find the range of values of k for which the line and the parabola do not intersect. aie) eS WeNaL= ed 1. Linear simultaneous equations can be solved using elimination or substitution. 2 Simultaneous equations with one linear and one quadratic equation can have up to two pairs of solutions. You need to make sure the solutions are paired correctly. 3. The solutions of a pair of simultaneous equations represent the points of intersection of their graphs. 4 Fora pair of simultaneous equations that produce a quadratic equation of the form axe +bx+e=0 + b—4ac>0 two real solutions + =4ac=0 one real solution + B—4ae<0 — noreal solutions 5 The solution of an inequality is the set of all real numbers x that make the inequality true. 6 Tosolve a quadratic inequality + Rearrange so that the right-hand side of the inequality is 0 + Solve the corresponding quadratic equation to find the critical values + Sketch the graph of the quadratic function + Use your sketch to find the required set of values. 7 The values of x for which the curve f(a) < glx). The values of x for which the curve 3 f(x) > glo). y'< f(x) represents the points on the coordinate grid below the curve > f(x) represents the points on the coordinate grid above the curve y f(x) is below the curve y = g(x) satisfy the inequality (x) is above the curve y = g(x) satisfy the inequality = fx). 0. 9 IFy> f(x) or y< f(a) then the curve y= f(x) is not included in the region and is represented by a dotted line. IF y = f(x) or y = f(x) then the curve y = f(x) is included in the region and is represented by a solid line. After completing this chapter you should be abl © Sketch cubic gra intersection points of graphs te © Translate graphs Stretch graphs of unfamiliar functions gue eed 1 Factorise these quadratic expressions: a t+ 6xt5 bem ixt € International GCSE Mathematics graph of ypexdexn ‘+ International GCSE Mathematics 4 Solve each pair of simultaneous equatior a y=ex b y=x? xty=6 y=2x-1 € Sections 3.1,3.2 simpler function reflections and translations. Particle physicists comp: with transformations of know ns to determine Cm aise} CES UUs Sy @o ‘Accubic function has the form f(x) = ax’ + bx? + ex +d, where a, b, c and d are real numbers and ais non-zero. graphs The graph of a cubic function can take several different forms, depending on the exact nature of the function. Forthese two functions «is positive. For these two functions wis negative * IF pis.a root of the function f(x) then the graph of y= f(x) touches or crosses the x-axis at the point (p, 0). You can sketch the graph of a cubic function by finding the roots of the function. xx Sketch the curves with the following equations and show the points where they cross the coordinate axes. a y= 21-9 +8) by= xt Ixr+2) Sore, y= (— px— @lx—1) where p,q and are constants using technology. a ya(e-20-a +x) So the ve crosses the x-axis at (2,0), 1, 0) and (+4, O). 19 the y-axis at (0, ~2), When x =O, y= xixts So the curve cross Sox=O,x tor CEP URL EUSUUy Cais So the curve crosses the x-axis at (0, 0), 1, 0) ana (-2, 0} Sketch the following curves: =1P6 +) by © ye(e-2F x — Fe + 1) So the curve crosses the x-axis at (1, 0) and touches the x-axis at (1 Ol = xt? - 2-3) (x — 3x + 1) (x - Six + 1) Sox=O,.x=30r So the curve crosses the x-axis at (0, 0}, (3, 0) and (1, 0). ee 7S apron arse} e ya (e-ap O= wa" Se at (2, 0) only. and the curve crosses the x-axi9 PUP UDG g Ul} Sketch the curve with equation y= (x= 2 +x +2). ys (x Mhe +x 2) O= w= th? x42) So x-axis at (1, 0). only and the curve er When x = y= C2) =-2 So the curve crosses the y-axis at (0, -2). | ———— x CED A cui GED vou havent gorenoveh information to know the exact shape of the graph. It could also bbe shaped like this: a's 1 Sketch the following curves and indicate clearly the points of intersection with the axes: x= 3)00-2)or+ 1) x + I(x + 2x + 3) © yale 200 34-9) & yaxtet Ie I) i ys(e-2)Q2x- NQx+ 1) b ya(x= Devt 2)or+3) @ ya (vt I-90 +3) f yexte- 200+) bh ysate+ DU-x) J y=aQx—1or+ 3) COUEe UPR VSO Ul cy Cais 2 3 4 Sketch the curves with the following equations: a ya(r+ De=1) b ya(r+2x= 1? © yaQ2=xie4 @ y= (e-2)(e4 1)? € y= r+) f y=r -1)'x g y= (LaF +x) bh y=(x-13G-a) i y=x@-x) jo vexe-2) Factorise the following equations and then sketch the curves: a yaxtxt-2v b yas Sxt dy ce yaxte dex f yex-e i y=at-9n Sketch the following curves and indicate the coordinates of the points where the curves cross the axes: ‘The graph of y where >, eand dare real constants, x + bx? + ex + dis shown opposite, a Find the values of 6, eand d. (marks) b Write down the coordinates of the point where the curve crosses the y-axis, (1 mark) Ga ed Start by writing the equation in the form y= (x pix -@)(v—1). ‘The graph of y = ax* + bx? + cx + dis shown opposite, where a, b, ¢ and dare real constants, Find the values of a, b, and d. (Amarks) Given that f(x) = (x= 10)(x2 = 2x) + 12 a Express f(x) in the form x(ax? + bx + c) where a b and c are real constants. (3 marks) 'b Hence factorise fix) completely. (2 marks) © Sketch the graph of y = f(x) showing clearly the points where the graph intersects the axes, (Smarks) GC Ce aLair} CES UUs Sy @ Reciprocal graphs You can sketch graphs of reciprocal functions such as y=,» 4 and y = ~2 by considering their asymptotes. * * = The graphs of y= and y = 4 where k isa real EEE sn asymptote is tine which x constant, have asyrnptotes otxx= Oand y=, the graph approaches but never reaches. I vithk>0 EE) ED swerse Sketch on the same diagrai 12 ¥ a y=Sand COUEe UPR VSO Ul cy Cais EXE mcrae 1 Usea separate diagram to sketch each pair of graphs: 2 b y=Sandy 2 x g x ©) Points of intersection You can sketch curves of functions to show points of intersection and sol sto equations. = The x-coordinate(s) at the point(s) of intersection of the curves with equations y= f(x) and y= g(x) are the solution(s) to the equation f(x) = g(x). ex) a On the same diagram, sketch the curves with equations y = x(x ~ 3)and y= 2(1— x), b Find the coordinates of the points of intersection. Cs) arr} PUP UDG g Ul} b From the graph there are three points where the curves cross, labelled A, B and C. The x-coordinates are given by the tions to the equation OM ©) TED rovers soume On the same diagram, sketch the curves with equations y = x2(3x - a) and y where « and b are positive constants. - b State, giving a reason, the number of real solutions to the equation x°(3x - a) - 2 =0. Stn pica ED RENN RTROAY SPOS You can sketch curves involving unknown of intersection of the curves, This means constants. You should give any points of intersection with the coordinate axes in terms of there are only two values of x where the constants where appropriate. rca i : Dees pre einie eee es arses or x48x-a) So this equation has two real solutions. COUEe UPR VSO Ul cy a Sketch the curves y= and y = x23) on the Cais same axes, Using your sketch, state, with a reason, the number of real solutions to the equation. hy = xe - 3) b There is a single point of intersection so the 4 equation x(x ~ 3) = has one real solution Rearranging} wiy-3)=4 w(y-3)-420 So this equation has one real solution. ETD ernaen In each case: i sketch the two curves on the same axes fi state the number of points of intersection Problem-solving ‘Set the functions equal to each other to form an ‘equation with one real solution, then rearrange the equation into the form given in the question. fii_ write down a suitable equation which would give the x-coordinates of these points, (You are not required to solve this equation.) ay x2 = 1) @ y= -x,= gB y=xtr=4) iv a On the same axes, sketch the curves given by y =x ~ 3) and y= b Explain how your sketch shows that there are x= 3)=2, a On the same axes, sketch the curves given by y= (x-+ I)and y= b x4 6x+1=0. x+ Ior= IP only two real solutions to the equation 3x(x =D). Explain how your sketch shows that there is only one real solution to the equation Cc Ce arairy PUP UDG g Ul} 4a Onthe same axes, sketch the curves given by y= 4 and y= —x0r — 1). b Explain how your sketch shows that there are no real solutions to the equation L+er= Ps Sa On the same axes, sketch the curves given by y = 3% — a) and y e where a and b are both positive constants, (5 marks) b Using your sketch, state, giving a reason, the number of real solutions to the equation xt = ax*= =0. (mark) Goce Even though you don't know the values of a and +, you know they are postive, so you know the shapes of the graphs. You can label the point a €on the x-axis on your sketch of y =.x2(x~ a). © 6 a On the same set of axes, sketch the graphs of Sandy 23x47. (3 marks) bb Write down the number of ral solutions to the equation = 3x-+7 (1 mark) € Show that you can rearrange the equation to give (x + 1)(x + 2)@x=2) =0 (2 marks) @ Hence determine the exact coordinates of the points of intersection. (3 marks) 7 a On the same axes, sketch the curve y= x3 ~ 3x?—4y and the line y = 6x b Find the coordinates of the points of intersection. x? I(r 2) and the line y = 14 +2. ® 8 a Onthesame axes, sketch the curve b Find the coordinates of the points of intersection. ® 9 2 Onthe same axes, sketch the curves with equations y 2)(x + 2P and y ==»? -8. b Find the coordinates of the points of intersection. 10 a Sketch the graphs of y = x2 + 1 and 2y 1 (3 marks) b Explain why there are no real solutions to the equation 2x? —x + 3 = 0. (2 marks) © Work out the range of values of a stch that the graphs of y=? + @ and 2y = x1 have two points of intersection. (5 marks) COUEe UPR VSO Ul cy Cais @& Taanslating graphs You can transform the graph of a function by altering the function, Adding or subtracting a constant ‘outside’ the function translates a graph vertically. = The graph of y= flx) + a isa translation of the graph y= fx) by the vector (2). ‘Adding or subtracting a constant ‘i ide’ the function translates the graph horizontally. = The graph of y = f(x + a) isa translation of the graph y = f(a) by the vector (9). (0) + Lisa translation (9), or 2 unit in the direction of the postive y-axs y= 21s tantaton (2), o2untsin the diecion of the negative at S4S2aA7 Tea Sketch the graphs of: by x-2P eyset2 Co) Raia GRAPHS AND TRANSFORMATIONS eneRerarion fae a(x) = x0v=2) Sketch the following graphs, indicating any points where the curves cross the axes: a yefe+l) b y=aer+t) a The graph of Kal is So the graph of » = glx + 1) is Ke tt) CS UP RL LUC UUs} Cais C) * When you translate a function, any asymptotes are also translated. Amo Given that h(x) = 4, sketch the curve with equation y = h(x) + 1 and state the equations of any asymptotes and interseetions with the axes. The graph of y = hex) is The curve crosses the x-axs once. hye t= tet 1 tat So the curve intersects the x-axis at (1, 0) The horizontal asymptote is y= 1 The vertical asymptote is x =O. 1 Apply the following transformations to the curves with equations if) fay=29 iit y= 4 Tn each ease state the coordinates of points where the curves cross the axes and in if state the equations of the asymptotes. afe+2) b y+? © fly=1) @ fixy)-1 © fix -3 f fix-3) me arse} CES UUs Sy 2 a Sketch the curve) b On separate diagrams, sketch the graphs of: i y=f(x-+2) fi y=flx) +2. € Find the equations of the curves y = f(x + 2) and y= f(x) + 2, in terms of x, and use these equations to find the coordinates of the points where your graphs in part b cross the y-axis, x) where f(x) = (x= I+ 2, 3a Sketch the graph of y = f(x) where f(x) = 3°(I =x). b Sketch the curve with equation y = fix + 1. © By finding the equation f(x + 1) in terms of x, find the coordinates of the point in part b where the curve crosses the y-axis. 4a Sketch the graph of y = f(x) where f(x) = xe — 2). b Sketch the curves with equations y= f(x) + Zand y= flx+ 2). © Find the coordinates of the points where the graph of y= f(x + 2) crosses the axes. 5 a Sketch the graph of y= f(x) where f(x) = a(x 4). = fox + 2)and y= fla) +4, ¢ Find the equations of the curves in part b in terms of xr and hence find the coordinates of the points where the curves cross the axes. b Sketch the curves with equations y © 6 The point Pt4, -1) lies on the curve with equation y = f(x). a State the coordinates that point P is transformed to on the curve with equation f(x) (1 mark) b State the coordinates that point P is transformed to on the curve with equation pate. (1 mark) 7 The graph of y = fix) where f(x 0 Write down the equation for the transformed function in the form y= translated so that the asymptotes are at x = 4 and (3 marks) a @® 8 a Sketch the graph of y= x°~ 5x° + 6x, marking clearly the points of intersection with the axes. b Hence sketch y = (x~2)°~ 5(v=2)? + 60-2), 9 @ Sketch the graph of y=. + 4x? 44x. (6 marks) b The point with coordinates (-1, 0) lies on the curve with Look at your sketch and equation y =(x +a) +4(c +a)? + 4(e-+ a) where ais.a picture the curve siding constant. Find the two possible values of a. (3 marks) to the left or right. COUEe UPR VSO Ul cy Cais 1. The point Q-5, ~7) lies on the curve with equation y= f(x) ‘a State the coordinates that point Q is transformed to on the curve with equation y = flx + 2)—5. bb The coordinates ofthe point @ on a transformed curve are (-3,~6). Write down the transformation in the form y= fix +a) —b @©) Stretching graphs Multiplying by a constant ‘outside’ the function stretches the graph vertically. = The graph of » = af(x) isa stretch of the graph y= f(x) by @ scale factor of a in the vertical direction. y= 2f(x) isa stretch with scale factor 2 in the y-direction. All y-coordinates are doubled. y=) isa stretch with scale Factor din the }yirection. ll y-coordinates are halved Multiplying by a constant ‘inside’ the function stretches the graph horizontally. = The graph of y = (ax) isa stretch of the graph y= f(x) by a scale factor of 2 in the horizontal direction. =a) isa stretch with scale factor Sin the ‘-diecton.Allx-coordinates ae halved. f(x) isa stretch with scale factor’ in the x-direction. All x-coordinates are tripled n Cee aT} PUP UDG g Ul} Given that f(x) = 9 — 3°, sketch the curves with equations: a y=f2x) by =2fs) a i= 9-48 So fix) = - 0G +9). The cuveis y=(3-xI3+x) 0=G-sG +) ———+ Sox=3orx=-3 So the curve crosses the x-axis at (3, 0) and (+3, 0). When x =O, y= So the curve crosses the y-axis at (0, 2) x3=9 The curve y = f(x) is y 2 5 O (2x) s0 the curve is 2 b y= 2itx) so the curve is 18 ons Pm II CEP URL EUSUUy Cais a Sketch the curve with equation y =. b On the same axes, sketch the curves x= r+ 1). (2x — 22x + 1) and y pa xix 2)ie + 1) r ‘ -»-- a. + | ¥ Eee You need to work out the relationship between ‘each new function and the original function. H.xbe~2)0¢ + 1) = fox) then 2x2x-2\@x +1) = fx), and x(x r+ 1) =f), is, = The graph of y = f(x) is a reflection of the graph of y= f(a) in the x: = The graph of y = f(-x) is a reflection of the graph of = f(x) in the y-axis. Example @§) (On the same axes, sketch the graphs of y = f(x), CMa GRAPHS AND TRANSFORMATIONS ENED uss 1 Apply the following transformations to the curves with equations y = f(x) where: i foo i fl) = ii A=} In each case show both f(x) and the transformation on the same diagram. a f(x) b fix) © fx) d fl4x) e fx) f 2K) g fo) h atx) i fey, i He) 2 a Sketch the curve with equation y = f(x) where fs) =°-4. ID part by reartange b Sketch the graphs of » f(x). “the second equation into ‘the form y = 3f(x) 3 a Sketch the curve with equation y = flx) where f(x) = (x = 2)(x + 2). b Sketch the graphs of. 5x). y= f(2x) and y = ~flx). @® 4 4 Sketch the curve with equation y = %(6~3) b On the same axes, sketch the curves with equations: Let f(x) = x26 -3) and try to y= (2xPQx-3) fi y=—x%r-3) write each of the equations in part b in terms of fx). 5 a Sketch the curve y= x2 + 3x4. b On the same axes, sketch the graph of Sy-=2? + 3x-4. ketch the graph of y= x2(x— 2 b On the same axes, sketch the graph of 3y= (x 2F. x). © 7 The point P(2, ~3) lies on the curve with equation » a State the coordinates that point P is transformed to ‘on the curve with equation y = f(2x), (1 mark) b State the coordinates that point P is transformed to on the curve with equation y = 4f(x), (1 mark) © 8 The point O-2, 8) lies on the curve with equation y State the coordinates that point Qis transformed to oon the curve with equation y = fix). (1 mark) 9 a Sketch the graph of y = (x = 2)(x = 3). (4 marks) b The graph of y = (ax ~2)(ax ~ 3) passes through the point (1, 0). Find two possible values for a. (3 marks) Cars 1. The point RG, -6) lies on the curve with equation y = f(x). State the coordinates that point R is transformed to on the curve with equation 2 The point Si-4, 7)is transformed to a point $'(-8, 175). Write down the transformation in the form y = afibx. CEP URL EUSUUy Cais @ Transforming functions You can apply transformations to unfamiliar functions by considering how specific points and features are transformed. ‘The diagram shows a sketch of the curve fix) which passes through the origin. The points A(1, 4) and B(, 1) also lie on the curve. Sketch the following: fi+l) — b yeflx-1 © yeti) -4 fo) e y-1=f) In cach case you should show the positions of the images of the points O, A and B. a ite?) 5 cen aie} CES UUs Sy 1 The following diagram shows a sketch of the curve with equation y = f(x). The points A(0, 2), BU, 0), (4, 4) and D(6, 0) lie on the curve. Sketch the following graphs and give the coordinates of the points 4, B, C and D after each transformation: afi+D bd fy-4 eft) d f(x) © 3x) f fy Pat) hf 2 The curve y = f(x) passes through the origin and has horizontal asymptote y = 2 and vertical asymptote x= 1, as shown in the diagram, Sketch the following graphs. Give the equations of any asymptotes and give the coordinates of intersections with the axes after each transformation. afiy+2 biel) € 2x) d f{x)-2 e f(x) f fx) h fox) COUEe UPR VSO Ul cy Cais 3 ‘The curve with equation y = f(x) passes through the points A(—4, ~6), B{-2, 0), C10, =3) and D(A, 0) as shown in the diagram, Sketch the following and give the coordinates of the points 4, B, Cand D after each transformation, a fix 2) b flxy+6 © f(x) d fiv+4) e fixy+3 f 3fx) @ Hi) h fx) i -ftx) i fiey A sketch of the curve y= f(x) is shown in the diagram, The curve has a vertical asymptote with equation x = —2 and a horizontal asymptote with equation y = 0. The curve crosses the y-axis at (0, 1). a Sketch, on separate diagrams, the graphs of: i 2) re) fix=2) W foy-1 vf, wi fo) In each case state the equations of any asymptotes and, if possible, points where the curve cuts the axes, b Suggest a possible equation for f(x). The point PQ, 1) lies on the graph with equation y= f(x). a On the graph of y= flax), the point P is transformed to the point Q(4, 1). Determine the value of «. (mark) b Write down the coordinates of the point to which P maps under each transformation. i fx-4) ii 34x) xy-4 (3 marks) The diagram shows a sketch of a curve with equation y = f(x). The points A(-1, 0), B(0, 2), C(1, 2) and DQ, 0) lie on the curve. Sketch the following graphs and give the coordinates of the points B, Cand Dafier each transformation ayt2=fi) b Sy=iho © y-3=fe) dh Rearrange each equation 1109 into the form y= 7 me aie GRAPHS AND TRANSFORMATIONS os 1 a Onthe same axes, sketch the graphs of = x°(x ~ 2) and y= 2x— By solving a suitable equation, find the points of intersection of the two graphs. @® 2 4 Onthesame axes sketch the curves with equations y= and y= 1+. bb The curves intersect atthe points 4 and B. Find the coordinates of 4 and B. € The curve Cwith equation y= "+ pq, where p and q are integers, passes through A and B. Find the values of p and g. @ Add Co your sketch. 3. The diagram shows a sketch of the curve y= f(x). ‘The point B(0, 0) lies on the curve and the point 4(3, 4) sa maximum point. The line y = 2is an asymptote. Sketch the following, and in each case give the coordinates of the new positions of 4 and Band state the equation of the asymptote: a f2x) b Sho, © x2 d fv+3) e fix 3) f yet © 4 The diagram shows the curve with equation y=5+2y— 2 and the line with equation y ‘The curve and the line intersect at the points Aand B. Find the x-coordinates of 4 and B. (4marks) © 5 The diagram shows a sketch of the curve with equation y= £3), On separate axes, sketch the curves with equations: fiw) (marks) fx) (2 marks) Mark on each sketch the x-coordinate of any point, or points, where the curve touches or erosses the x-axis. COUEe UPR VSO Ul cy Cais &® 6 The diagram shows the graph of the quadratic Function f(x). The graph meets the x-axis at (1, 0) and (3, 0) and the ‘minimum point is (2, -1). a Find the equation of the graph in the form Year thr te. (2 marks) b On separate axes, sketch the graphs of iy=te+2) ii y=f2x). (2 marks) € Oneach graph, label the coordinates of the points at which the graph meets the x-axis and label the coordinates of the minimum point. fx) = (= Dow= or De a State the coordinates of the point at which the graph y = f{x) intersects the y-axis, (1 mark) b The graph of y= af(x) intersects the y-axis at (0, ~4), Pind the value of a (mark) © The graph of y= f(x +) passes through the origin, Find three possible values of b, (3 marks) The point P(4, 3) lies on a curve with equation y= f(x). a State the coordinates of the point to which P is transformed on the curve with equation: y= f(3x) x5) iv -y= fe) v +2) b Pis transformed to point (2, 3). Write down two possible transformations of f(x). is transformed to point (8, 6). Write down a possible transformation of f(x) if i 109 is translated only f(x) is stretched only. a Factorise completely x° - 6x? + 9x (2 marks) b Sketch the curve of y = x ~ 6x2 + 9x, showing clearly the coordinates of the points where the curve touches or crosses the axes, (4marks) © The point with coordinates (4, 0) lies on the curve with equation y= (= AP = 6r= A)? + 9(or~ A) where kis a constant. Find the two possible values of k (G marks) N(x) = xr 2° Sketch, on separate axes, the graphs of: a yet) (2 marks) b y=fr+3) (marks) Show on each sketch the coordinates of the points where each graph crosses or meets the axes Given that lx) =4.x 40 a Sketch the graph of = fix) ~ 2 and state the equations of the asymptotes, (3marks) b Find the coordinates of the point where the curve y = f(x) ~2 cuts a coordinate axis. (2 marks) © Sketch the graph of y= fix +3). (marks) @ State the equations of the asymptotes and the coordinates of the point where the curve cuts a coordinate axis. (2 marks) 79 Cm arse} PUP UDG g Ul} Challenge The point K(6,—4) lies on the curve with equation y = f(x) State the coordinates that point 2 is transformed to on the curve with equation y = flx + ¢)~d. BT ed 1 If pisa root of the function fix), then the graph of y= the point (p, 0). (x) touches or crosses the x-axis at k here Kis a real constant, have asymptotes at x= O and y=0. 2. The graphs of » E and y 4 3. The x-coordinate(s) at the point(s) of intersection of the curves with equations y= f(x) and 37 = g(x) are the solution(s) to the equation f(x) = g(x). 4 The graph of, fa) by the vector (°), (x) + ais a translation of the graph 5 The graph of y= f(x + a) isa translation of the graph y= f(x) by the vector (6) 6 When you translate a function, any asymptotes are also translated. 7 The graph of y= afl) i in the vertical direction. a stretch of the graph y= flx) by a scale factor of a See tn aeeere teenyer ereemm aren in the horizontal direction. 9 The graph of » =—f(x) is a reflection of the graph of y= x) in the x-axis, 10. The graph of y= f{-x) isa reflection of the graph of y = f(x) in the y-axis. Palast as Review exercise Do not use your calculator for this question. 7 Do not use your calculator for this question. a Write down the value of 84 a b Find the value of 8. Q «Section 1.4 Do not use your calculator for this question, a Find the value of 1257 Q) b Simplify 24x? + 18, Q «Sections 1.1, 14 Do not use your calculator for this question, a Express \80 in the form a5, where «is an integer. b Express (4 — 3) in the form b + «3, where 6 and c are integers, @ @ © Section 15 Do not use your ealeulator for this question, a Expand and simplify (44304 - 03). @ Express a in the form a + bv3, where a and b are integers @ Sections 1.5, 1.6 Here are three numbers: 1-Vk 24 SVK and WE Given that k is a positive integer, find: a the mean of the three numbers (2) b the range of the three numbers. a Section 1.5 1 Given that y= :x", express each of the following in the form kx", where k and 1 are constants. ay! b sy a a «Section 1.4 Find the area of this trapezium in em: Give your answer in the form a + by2, where a and b are integers to be found. (4) «Section 1.5 (3+ Whom> (5+ 33) em: © 8 Do not use your calculator for this question. Given that p = 3 - 2/2 and q = 2 - v2, pea find the value of = Give your answer in the form nr + 1/2, where m and n are rational numbers to be found. @ Sections 1.5,1.6 a Factorise the expression P= 10x +16, @ b Hence, or otherwise, solve the equation 2) — 10(89) + 16 Q # Sections1.3,2.1 x?- 8x - 292 (x +a)? +b, where aand 5 are constants. a Find the value of @ and the value of b. b Hence, or otherwise, show that the roots of x2 = 8-29 = 0 are c+ 5, where ¢ and dare integers, 8) + Sections 2.1, 2.2 @ @u &® 2 ®©3 & 15 ‘The functions f and g are defined as f(x) = x(x = 2) and g(x) = ¥45,¥ER, Given that fa) = g(a) and a > 0, find the value of a to three significant figures, a #Sections2.1,2.3 a Given that f(x) 6x4 18,220, express f(x) in the form (x ~ a)? +B, where a and b are integers The curve C with equation y = f(x), x= 0, meets the y-axis at P and has a minimum point at 0. b Sketch the graph of C, showing the coordinates of P and Q. ‘The line y = 41 meets C at the point R. ¢ Find the x-coordinate of R, giving your answer in the form p + qi2, where p and g are integers. Q ‘© Sections 2.2, 246 24 QDe + k has @ @ ‘The function h(x) equal roots. a Find the value of k. a b Sketch the graph of y = h(x), clearly labelling any intersections with the coordinate axes, @ Sections 1.5,2.4,2.5, The function g(x) is defined as a(x) = 99 - 7x8 - 80, VER, a Write a(x) in the form x'(x" + a(x" +B), where a and b are integers, @ b Hence find the three roots of a(x). (1) + Section 2.3, Given that xo 10x + 36E (x +a) +h, where a and b are constants, a find the value of @ and the value of. @ b Hence show that the equation x2 + 10x +36 =0 has no real roots, (2) The equation x° + 1l0x+ k=0 has equal roots. fala estat Find the value of k. For this value of k, sketch the graph of y= x? + 10x + k, showing the coordinates of any points at which the graph meets the coordinate axes, @ @) # Sections 2.2, 24, 2.5, Given that x° 420432 (x-+a)?+h, a find the value of the constants aand b. Q b Sketch the graph of y 2x43, indicating clearly the coordinates of any intersections with the coordinate axes, 2) © Find the value of the discriminant of 3° +2v+ 3. Explain how the sign of the discriminant relates to your sketch in part b. Q The equation x°-+ kx +3=0, where k isa constant, has no real roots. Find the set of possible values of k, giving your answer in surd form, show that M44y-8=0. b Hence, or otherwise, solve the simultaneous equations: yex-4 @) 2x =x, giving your answers in the form a+ 5\3, where aand b are integers. (4) + Section3.2 Find the set of values of x for whieh a 3Qx+1)> 5-26 Q b 2x?-7x43>0 @ © both 32x + 1) > 5~2vand 2x? 7x +350. wo + Sections 3.4,3.5 Palast as &® v &® 1 ‘The functions p and q are defined as p(x) = -20¢ + 1) and q(x) =? - Sx +2, x € R, Show algebraically that there is no value of x for which p(x) = q(1) @ # Sections2.3,2.5 a Solve the simultaneous equations: yt2n=5 2x? 3x-y=16 6 b Hence, or otherwise, find the set of values of x for which: 2x? - 3x - 16 > 5 - 2x Q © Sections 3.2, 35 ‘The equation x2 + kx + (k +3) =0, where kis aconstant, has different real roots a Show that k? — 4k ~ 12> 0. 2 b Find the set of possible values of k. (2) # Sections2.5,3.5 Find the set of values for which 6 x+5 <2,x#-5 © © Section 3.4 The functions f and g are defined as f(x) =9 and g(x) = 14 - 6x, ER. a On the same set of axes, sketch the graphs of y= f(x) and y = g(x). Indicate clearly the coordinates of any points where the graphs intersect with each other or the coordinate axes. © b On your sketch, shade the region that satisfies the inequalities y > 0 and fx) > gt. a + Sections 3.2,3.3,3.7 a Factorise completely x° — 4x w b Sketch the curve with equation 2 — 4x, showing the coordinates Of the points where the curve crosses the x-axis, 2 ¢ Ona separate diagram, sketch the curve with equation ye(v=1S=40r- 1) showing the coordinates of the points where the curve crosses the a-axis. (2) «Sections 1.3, 61,45 @©% The figure shows a sketch of the curve with equation y= f(x). The curve crosses the x-axis at the points (2, 0) and (4, 0). The ‘minimum point on the curve is PG, ~2). In separate diagrams, sketch the curves with equation a y=—fx) Q) b y=f2x) 2 On each diagram, give the coordinates of the points at which the curve crosses the x-axis, and the coordinates of the image of P under the given transformation. + Sections 4.5, 4.6 ‘The figure shows a sketch of the curve with equation y= f(x). The curve passes through the points (0, 3) and (4, 0) and touches the x-axis at the point (1, 0). On separate diagrams, sketch the curves with equations a y=fe+)) @ b y=2f) Q) © y=flyx) @ On each diagram, show clearly the coordinates of all the points where the curve meets the axes. © Sections 4.4, 45, 4.6 @©u Given that f(x) = 5." # 0, a sketch the graph of and state the equations of the asymptotes Q b find the coordinates of the point where y= fix) +3 crosses a coordinate axis. 2 Sections 42,44 ‘The point (6, -8) lies on the graph of (x). State the coordinates of the point to which P is transformed on the graph with equation: f(x) oO f(x -3) wo © ) a + Section 4.6 The curve C, has equation y= -%, where is a positive constant. The curve C, has equation y= (x — 6), where 6 is a positive constant. a Sketch Cand C, on the same set of axes. Label any points where either curve meets the coordinate axes, aiving your coordinates in terms of aand 6. w b Using your sketeh, state the number of real solutions to the equation x(v= 5? =-7. a Sections 4.2, 4.3 30 a Sketch the graph of y= fala estat showing clearly the coordinates of the points where the curve crosses the coordinate axes and stating the equations of the asymptotes. (4) » Thecurve with equation y= <> ~4 passes through the origin, Find the two possible values of k. @ + Sections 4.2, 46, 46 ELD BED err nen 1a Solve the equation x? ~ 10x + 9 =0. bb Hence, or otherwise, solve the equation 32Q'— 10) =-1. «Sections 1.1,13,21 2 Arectangle has an area of 6cm? and a perimeter of 8/2 em. Find the dimensions of the rectangle, giving your answers as surds in their simplest form. + Sections 1.5,22 3. Show algebraically thatthe graphs of Y=3e +x? —wand y= 2xbe— Dex +1) have ‘nly one point of intersection, and find the coordinates ofthis point. + Section 33 ASL Cae en eee ‘After completing this unit you should! be able to: © Calculate the gradient of a line joining a pair of points > pages a6-87 © Understand the link between the equation ofa line : and its gradient and intercept > pages 87-89 Find the equation of a line given () the gradient and ‘one point on the line or (if) two points on the line ~> pages 89-92 know and use the rules for parallel and perpendicular gradients ~ pages 93-96 © Find the point of intersection for a pair of straight lines -» pages 97-09 © Solve length and area problems on coordinate grids. > pages 96-99 Cegece ec’ 1. Find the point of intersection of each pair of lines: =4x+Tand5y=2x-1 x-Land3x+Ty=11 Land 5x—Ty=14 + International GCSE Mathematics J Hl 2 Simplify each of the following: a 80 b 200 ¢ 125 3 Make y the subject of each equation: ¢ 2x-5p= Alandscape architect can use a straight-line graph to estimate how long it will take fora tree to grow to ven height if it continues at the a 6x+3y—15=0 b 2x—5y-9=0 current rate. CM aie} SASL} ED y-nsve You can find the gradient of a straight line joining two points by considering the vertical distance and the horizontal distance between the points. 1 The gradient m of a line joining the point with coordinates (x3) to the point with coordinates (x,»2) can be calculated using the formula nt ex - formula using GeoGebra. Work out the gradient of the line joining (-2, 7) and (4, 5), (4,5) EIEN 2) BLED wermneraow The line joining (2, 5) to (4, a) has gradient ~1, Work out the value of a. ae een Sous 1 Work out the gradients of the lines joining these pairs of points: ts =~ 6 a (4,2), (6,3) b 19,64) © (-4,5),(1,2) 4 (2,-3), (6.5) © (-3,4),(7,-6) £ (-12,3),(-2,8) 4), (10, 2) bG.2.G.4) i GD) J (24,96), 0,0) k (13,22), (8.8, -4.7) 1 @,5a), (10a, 0) 1m (36, -26), (7b, 2) n (p.P?). (GP) Senet uaa sy Crary 2. The line joining (3, -5) to (6, a) has gradient 4. Work out the value of a. 3. The line joining (5, b) to (8, 3) has gradient ~3. Work out the value of 6. 4 The line joining (c, 4) to (7, 6) has gradient }. Work out the value of «. 5 The line joining (-1, 2d) to (1, 4) has gradient ~ 5. Work out the value of d. 6 The line joining (~3, -2) to (2e, 5) has gradient 2, Work out the value of e. 7 The line joining (7, 2) to (f, 3f) has gradient 4. Work out the value of f. 8 The line joining (3, -4) to (~g, 2g) has gradient ~3. Work out the value of g. ® 9% Show that the points 4(2, 3), B(4, 4) and Gcuena C(10,7) can be joined by a straight line. Find the gradient of the line joining the points and B, and the line joining the points A and C. @® 10 show hate pins 24,50, 0.40) EID ena tine alle and C(6a, a) are collinear, GB marks) "The same straight line. The equation of a straight line can be written in the form y= mex + ¢, where nris the gradient and cis the y-intercept. @ The equation of a straight line can also be written in the form ax + by + ¢ = 0, where a,b and c are integers. ‘Write down the gradient and y-intercept of these lines: a ye-3x+2 b 4x-3+5=0 — ze by-dee$ Gradient = 4 and y-intencert = (, (ane 1 Fi ed CM atin} SASL} Write these lines in the form ax + by + ¢= 0: apedxs3 byeras boas | b dr+y-5=20 ———___+ So P has coordinates (2, QJ 1 Work out the gradients of these lines: a ye-2e+5 bys-x47 ec yade3x d y=yx-2 eyes f ysive} @ 2v-dy+ h l0x-Sy+1= i -w+2y-4=0 j 3x4 6p47=0 k 4x4 2y-920 1 9x4 64220 2 These lines cut the y-axis at (0, ¢). Work out the value of cin each ease. a yeoxed by=2x-5 ec yehy- d y=-3x e ya$red f y=2-Tx g iw dy + 8= h 4x Sy-10= i -2x+y-9=0 j It4y+12=0 Kk Tx-2y +3=0 1 -Sx+4y+2 3° Write these lines in the form ax + by + ¢= 0. b y=3x-2 e y=fue2 2 wax42 Senet uaa sy Crary 5) The line 3x + 2)’ = 0 meets the x is at the point R. Work out the coordinates of R. 6 The line Sx — 4y +20 = 0 meets the y- Work out the coordinates of 4 and B. is at the point A and the x-axis at the point B. 7 Alline I passes through the points with coordinates (0, 5) and (6, 7). a Find the gradient of the line b Find an equation of the line in the form ax + by + ¢=0. © Aline [cuts the x-axis at (5, 0) and the y-axis at (0, 2). a Find the gradient of the b Find an equation of the line in the form ax + by + ¢=0. 9 Show that the line with equation ax + by +¢=0 GEES Ewereres ea % has gradient ~F and cuts the y-axis at =} In question 9, try solving a similar problem with numbers ist (1 mark) (2 marks) 8 © 10 The line / with gradient 3 and y-intercept (0, 5) has the equation ax 2y +.¢=0. Find the values of @ and c. Find the gradient and y-intercept of the straight line with equation 3x + Zy +2=0. (2 marks) 11. The straight line / passes through (0, 6) and has gradient ~2. It intersects the line with equation 5x ~ 8 - 15 = 0 at point P, Find the coordinates of P. (A marks) 12. The straight line J, with equation y= 3x ~7 intersects the straight line /, with equation @ @ ax + 4y — 17 = 0 at the point P(-3, 6). a Find the value of b. (1 mark) b Find the value of a. (2 marks) ‘Show that the equation of a straight line through (0, @) and (b, 0) is ax + by ~ ab =0. ©) Equations of straight lines You can define a straight line by giving: + one point on the line and the gradient + two different points on the line You can find an equation of the line from either Thisis the of these conditions. point on the tne = The equation of a line with gradient m that you know passes through the point with coordinates (xy, ,) can be written as y— y, = m(x—x,). ‘Thisis any point on the lin, Ce are} SASL} Find the equation of the line with gradient 5 that passes through the point (3, 2). y-2= 5-3) yo 2=Sx-15 5x - 13 CID 7 GON 7) EE weracinon Find the equation of the line that passes through the points (5, 7) and (3, -1). petedy-12 atieie= 2) — Eas EXE rc 1 Find the equation of the line with gradient m that passes through the point (x, y) when: b m= 3and (xs, 94) = (2,1) 2and (x91 Tand (x), 9) 2. Find the equations of the lines that pass through these pairs of points a (2,4) and (3, 8) b (0,2) and (3, 5) GED inecach case, © (-2, 0)and (2, 8) (5, -3) and (7, 5) find the gradient e (3-1 and (7,3) f (4,1) and (6,4) Cas g (1, -5) and (~3,3) h (4, =I) and (3, -9) aaa i G2andgd i Rand G3) © 3 Find the equation of the line ! which passes through the points 4(7, 2) and B(9, -8) Give your answer in the form ax + by + ¢ =0. (3 marks) 4 The vertices of the triangle ABC have coordinates (3, 5), B(-2, 0) and C(4, -1). Find the equations of the sides of the triangle. Senet uaa sy Crary G®) § The straight line passes through (a, 4) and, Ga, 3). An equation of lis x + 6y + ¢=0. Itis often easier to find unknown Find the value of a and the value of c. (marks) _veluesin the order they are piven ; inthe question. Find the value of a 6 The straight tine f passes through (7a, 5) and (3a, 3). pe ae tees An equation of lis x + dy ~ 12=0. Find the value of a and the value of b. (3 marks) Challenge Consider the line passing through points (x) and (1, 2). {a Write down the formula for the gradient, m, of the line, 'b Show that the general equation ofthe line can be written in the form m € Use the equation from part b to find an equation of the line passing through the Points (8, 4) and (7). Example @) ‘The line y = 3x — 9 meets the x-axis at the point A. Find the equation of the line with gradient 3 that passes through point 4. Write your answer in the form ax + dy +¢=0, where a, b and ¢ are integers, O=3x-9 80x = 3, A is the point (3, 0) o=hw-3) ape -2xt yt EX aD == Ss The lines y = 4x ~ 7 and 2x + 3y ~ 21 =0 intersect at the point A. The point B has coordinates (~2, 8) Find the equation of the line that passes through points A and B, Write your answer in the form ax + by +¢=0, where a, band care integers 2x + ax £7)- 21-0 2x + 12x ° tax = 42 x=3 y = 4@) ~ 7 =5 so A is the point 3, 5) ca aie Bea ar ay 1 The line y= 4x — 8 meets the x-axis at the point A. Find the equation of the line with gradient 3 that passes through the point A. 2 The line y = -2x +8 meets the y-axis at the point B. Find the equation of the line with gradient 2 that passes through the point B. 3. The line y = 4y + 6 meets the x-axis at the point C. Find the equation of the line with gradient 3 that passes through the point C. Write your answer in the form ax + by + ¢ = 0, where a, band care integers. @® 4 The line y = jx +2 meets the y-axis at the point B, The point C has coordinates (5, 3). Find the gradient of the line joining the points B and C. ees ‘A sketch can help you check whether ‘or not your answer looks correct. @® 5 The line that passes through the points (2, -5) and (-7, 4) ‘meets the x-axis at the point P. Work out the coordinates of the point P. ©® 6 The line that passes through the points (-3, -5) and (4, 9) meets the y-axis at the point G. Work out the coordinates of the point G. ©® 7 The line that passes through the points (3, 23) and (~1}, 4) meets the y-axis at the point J. Work out the coordinates of the point J. @® 8& The lines y =. and y = 2x ~ 5 intersect at the poit aradient } that passes through the point A. Find the equation of the fine with ©® 9% The lines y= 4x - 10 and y= x - 1 intersect at the point 7. Find the equation of the line with gradient -3 that passes through the point T. Write your answer in the form ax + by + ¢ where a, 6 and ¢ are integers ©® 10 The line p has gradient § and passes through the point (6, 12). The line g has gradient —1 and passes through the point (5, 5). The line p meets the y-axis at A and the line g meets the x: at B. Work out the gradient of the line joining the points A and B. ©® 11 Thetine point Q. @® 12 The line y= 3x ~ 5 meets the x-axis at the point M. The line y = point NV. Find the equation of the line joining the points M and NV. Write your answer in the form ax + by + ¢=0, where a, band eare integers. © 13 Thetine y=20— 10 meets the saxs at the point A. The line point B. Find the equation of the line joining the points 4 and B. 1+ 6 meets the x-axis at the point P. The line y = 3 ~-4 meets the y-axis at the ‘ind the equation of the line joining the points P and Q. 2x +4 meets the y-axis at the @® 14 The ine y =4y-+ 5 meets the yas atthe point C, The line » = -3v~ 15 meets the waxisat the point D. Find the equation of the line joining the points C and D. Write your answer in the form ax + by + c= 0, where a, b and care integers. @® 15 The lines y = x - 5 and y = 3x - 13 intersect at the point 8, The point T has coordinates (-4, 2). Find the equation of the Tine that passes through the points Sand 7. @® 16 The ines y= -20 + 1 and y-=.1+ 7imterseet atthe point L. The point M has coordinates (~3, 1). Find the equation of the line that passes through the points L and IM. Senet uaa sy Crary @ Parallel and perpendicular lines * Parallel lines have the same gradient. m Aline is parallel to the line 6x + 3y ~ 2= and it passes through the point (0, 3) Work out the equation of the i a -____— ae] The gradient of this line ip ~2. The equation of the line sy ETD sass 1 Work out whether or not each pair of lines is parallel. a ysSx-2 b Txt ldy-1=0 © 4x-3y-8=0 1 1Sx-3y +9=0 yetyt9 3x-4y-8=0 © 2 The line r passes through the points (1, 4) and (6, 8) and the line s passes through the points (5, -3) and (20, 9). Show that the lines r and s are parallel. ©® 3 The coordinates of a quadrilateral ABCD are A(-6, 2), BU, 8), aD ‘A trapezium has exactly. C6, 1) and D{-9, -8). Show that the quadrifateral is a trapezium, one pair of parallel sides, 4 A line is parallel to the line y = Sx +8 and its y-intercept is (0, 3). BD Tre line wil have Write down the equation of the line gradient 5. 5 A line is parallel to the line y = ~2y + 1 and its y-intercept is (0, ~4). Work out the equation of the line, Write your answer in the form ax + by + ¢= 0, where a, b and e are integers, © 6 A line is parallel to the line 3x + 6y + 11 = O and its intercept on the y-axis is (0, 7) Write down the equation of the line ® 7 A line is parallel to the line 2x - 3y Write down the equation of the fine O and it passes through the point (0, 0) 8 Find an equation of the line that passes through the point (~2, 7) and is parallel tothe line y = 4x + 1. Write your answer in the form ax + by + ¢=0. arse} SASL} Perpendicular lines are at right angles to each other. If you know the gradient of one line, you can find the vaifienk ot the other The shadled triangles . 7 are congruent. = Ifalinehasa gradient of m,aline 1. Line dy has gradient perpendicular to it has a gradient of 2. fem * IF two tines are perpendicular, the product Line has sracient of their gradients wees a Work out whether these pairs of lines are parallel, perpendicular or neither: a 3x-y-2=0 b yshx x+3y-6=0 2x-p+4=0 ax-2 50 yeax-2 The gradient of this line Is 3. The gradient of this line is 50 the lines are perpendicular as axepent The gradient of this Ine I ax-y44 axed Soy The gradient of this line is 2 x44 The lines are not parallel a5 tl fe oeren ED sein CF BY sila _ Using technology. ee Brea era Example @) A line is perpendicular to the line 2y — x ~ 8 = and passes through the point (5, ~7), Find the equation of the line, teed Fill in the steps of this problem yourself: See ee area peeae cerned eee eee pean ST eee ee rere ecuatonofthetine Rearranging, » Gradient of y EXD wos 1 Work out whether these pairs of lines are parallel, perpendicular or neither: 2 1 ayade+2 bysiv-l 3 Sy-yt4=0 j 4v-Sy+1s0 k 3x42) 1 Sx-y42 8x- 10y-2=0 v4 3y-6=0 2x4 Wy -4=0 2. A line is perpendicular to the line y Find an equation of the line. x 9-and passes through the point (0, 1). ® 3 Aline is perpendicular to the line 3x + 8y ~ 11 = 0 and passes through the point (0, 8), Find an equation of the line. 4 Find an equation of the line that passes through the point (6, -2) and is perpendicular to the line y = 3x + 5. 5 Find an equation of the line that passes through the point ( and is perpendicular to the line y= 3x +6. 3) © 6 Find an equation of the line that passes through the point (3, 4) and is perpendicular to the line 4x ~ 6 +7 = 0. 7 Find an equation of the line that passes through the point (5, ~5) and is perpendicular to the line y = 3x + 5. Write your answer in the form ax + dy + ¢= 0, where a, band ¢ are integers Ce arse} Seman 8 Find an equation ofthe fine that passes through the point (-2, -3) and is perpendicular to the line y = ~4y + 5. Write your answer in the form ax + by + ¢=0, where a, b and ¢ are integers. ® 9 The line passes through the points (~3, 0) and (3, -2) and the linen passes through the points (1, 8) and (—1,2). _Derft do more work than you need to, Show that the lines J and are perpendicular. You only need to find the gradients of both lines, not their equations ® 10 The vertices of a quadrilateral ABCD have coordinates ACI, 5), B71), C(S,-3) and D(-3, 1). ‘The sides ofa rectangle Show that the quadrilateral is a rectangle. are perpendicular. GP) 11 A line has equation 5x + Ly -7=0 and crosses the x-axis at 4. ‘The line fy is perpendicular to /, and passes through A. a Find the coordinates of the point 4. (1 mark) (B marks) b Find the equation of the line fy, Write your answer in the form ax + by +¢=0. axis and the point B lies on the x xis as shown in the diagram. G12 The points A and Clie on the 40,4) Problem-solving Sketch graphs in coordinate geometry problems ‘are not accurate, but you can use the graph to make sure that your answer makes sense. In this ‘question c must be negative. ‘The line through points A and B is perpendicular to the line through points B and C Find the value of ¢. (6 marks) G@® Length and area You can find the distance between two points 4 and B by Y a considering aright-angled triangle with hypotenuse 4B. tea = You can find the distance d between (x,,,) and (¥,,¥2) by using the formula d = (x — xy? + Ji)? Senet uaa sy Crary anrsis Find the distance between (2, 3) and (5, 7). e]7) o do (5-2 47-3" a4 ge a + a) \25 ‘The straight line /, with equation 4x - y =0 and the straight line /, with equation 2x + 3p - 21 = a Work out the coordinates of 4. b Work out the area of Ne AOB where Bis the point where /; meets the x-axis Le -21= a 2x + 34x) - 21 = aaa ° ax(3 intersect at point 4. a Equation of fs So point A has coordinates: ie ))|) EEE b The tangle AOB has a height ete ceca Tenet te omdhaestpoit A, 2x4 3y 21> arto eee The tangle AOB has a base length —~— aaa a units. Area CMa ait Bea ar ay sours 1 Find the distance between these pairs of points: a (0,1), (6,9) b (4, -6), (9, 6) © (3,1),(-1, 4) da ), (4,7) e (0, 4), (5, 5) f (-2,-7),(5, 2 Consider the points 4(-3, 5), B(-2,~2) and C(3,-7). IEE we tine segments are congruent Determine whether or not the fine joining points ae aeaeaes ene A and Bis congruent to the line joining points B and C. 3 Consider the points P(11, -8), O(4, -3) and R(7, 5). Show that the Tine segment joining the points P and Q is not congruent to the line joining the points Q and K @® 4 The distance between the points (—1, 13) and (x, 9) is 65. G@ISTEUES NETS Find two possible values of x. Use the distance formula to formulate a quadratic equation in x 5. The distance between the points (2, 6 a Show that the straight line /, with equation y = 2x +4 is Problem-solving and (5, 7) is 3/10. Find two possible values of y ® ® parallel to the straight line 4 with equation 6x - 3y-9 = 0. THETSNOTESST SET b Find the equation of the straight line f that is perpendicular between two parallel lines is. to A, and passes through the point (3, 10) the perpendicular distance ¢ Find the point of intersection of the lines /, and fy pelreomiely 4 Find the shortest distance between lines f, and é, 7 A point P lies on the line with equation y = 4 ~ 3x, The point Pisa distance V34 from the origin. Find the two possible positions of point P. (S marks) © 8 The vertices of a triangle are A(2, 7), BIS, -6) and C(8, -6). QEEREED scatene triangles have a Show that the triangle is a sealene triangle. three sides of afferent lengths. b Find the area of the triangle ABC. Draw a sketch and label the points 4, B and C. ae 9 The straight line /, has equation y = 7x ~ 3. The straight line /y has equation 4x + 3p ~ 41 = 0, The lines intersect at the point 4. a Work out the coordinates of A ‘The straight line /, crosses the x-axis at the point B. b Work out the coordinates of B. ¢ Work out the area of triangle AOB, where O is the origin. Senet uaa sy Crary 10 The straight line /, has equation 4x ~ Sy ~ 10 = 0 and intersects the x-axis at point The straight line /; has equation 4x ~ 2y + 20 = 0 and intersects the x-axis at point B. a Work out the coordinates of 4. b_ Work out the coordinates of B. The straight lines /, and /, intersect at the point C. € Work out the coordinates of C. Work out the area of triangle ABC. © 11 The points R(5, -2) and S(, 0) lie on the straight line /, as shown. a Work out an equation for straight line /, (2 marks) ‘The straight line /; is perpendicular to 4 and passes through the point R b Work out an equation for straight line bs, (2 marks) © Write down the coordinates of 7. (1 mark) 4 Work out the lengths of RS and TR leaving your answer in the form ky3 (2 marks) € Work out the area of ARST. (2.marks) @®) 12 The straight line /, passes through the point (~4, 14) and has gradient ~}. a Find an equation for j in the form ax + by +.¢=0, where a, band care integers, (3 marks) Write down the coordinates of A, the point where straight Hine /, crosses the y-axis, (1 mark) ‘The straight line /, passes through the origin and has gradient 3. The lines f, and J intersect at the point B. € Calculate the coordinates of B. (2 marks) 4 Calculate the exact area of AOAB. (marks) GE ©) IID cour rmenon 1 The straight line passing through the point P(2, 1) and the point Q(k, 11) has gradient ~ 7. a Find the equation of the line in terms of x and y only, (2 marks) b Determine the value of &. (2 marks) GB) 2 The points 4 and B have coordinates (k, 1) and (8, 24 — 1) respectively, where & is a constant. Given that the gradient of AB is + a show that k= 2 (2marks) b find an equation for the line through A and B. (3 marks) © 3 Theline £; has eradient + and passes through the point A(2, 2). The line L, has gradient -1 and passes through the point B(4, 8). The lines L, and £3 intersect at the point C. a Find an equation for Z, and an equation for Ly (4marks) b Determine the coordinates of C (marks) rari} SASL} © 4 a Find an equation of the line J which passes through the points A(1, 0) and (5, 6). (2 marks) ‘The Hine m with equation 2x + 3y = 15 meets /at the point C b Determine the coordinates of C. (marks) © _5§ Theline L passes through the points A(1, 3) and B(-19, ~19), Find an equation of L in the form ax + by + ¢=0. where a, and care integers. (3 marks) © 6 The straight tine J, passes through the points 4 and B with coordinates (2, 2) and (6, 0) respectively. a Find an equation of / (3 marks) ‘The straight line / passes through the point C with coordinate (~9, 0) and has gradient b Find an equation of / (2 marks) 7 The straight line / passes through A(1, 3,3) and B(2 + \3,3 +43). Show that / meets the x-axis at the point C{-2, 0) (marks) © _& The points 4 and B have coordinates (~4, 6) and (2, 8) respectively. A line p is drawn through perpendicular to AB to meet the y-axis at the point C a Find an equation of the line p. (3 marks) b Determine the coordinates of C (mark) B®) 9 The line /has equation 2 The line m passes through the point 4(0, 4) and is perpendicular to the line a Find an equation of m. (2 marks) b Show that the lines / and mr intersect at the point P(2, 3). (2 marks) The line n passes through the point B(3, 0) and is parallel to the line m, € Find the coordinates of the point of intersection of the lines / and n. (marks) B®) 10 The line /, passes through the points A and B with coordinates (0, -2) and (6, 7) respectively. The fine /; has equation x + y’=8 and cuts the y-axis at the point C. The Tine /, and ls intersect at D. Find the area of triangle ACD. (6 marks) © 11 The points A and B have coordinates (2, 16) and (12, ~4) respectively. A straight line /, passes through A and B. a Find an equation for /; in the form ax + by =e. (2 marks) The line fy passes through the point C with coordinates (-1, 1) and has gradient. b Find an equation for / (2 marks) Senet uaa sy Crary @&) 2 16 The points A(-1, -2), BU, 2) and C(k, 4), where kis a constant, are the vertices of AABC. Angle ABCis a right angle. a Find the gradient of AB. (1 mark) b Calculate the value of k (marks) ¢ Find an equation of the straight line passing through B and C. Give your answer in the form ax + by + ¢= 0, where a, band ¢ are integers. (marks) d Calculate the area of AABC, (2 marks) a Find an equation of the straight line passing through the points with coordinates (1, 5) and (4, -2), giving your answer in the form ax + by +¢=0, where a, b and c are integers (3 marks) The line crosses the x-axis at the point A and the y-axis at the point B, and O is the origin. b Find the area of AAOB. (marks) The straight line f, has equation 4y +x = 0. x= 3. The straight line /, has equation. a On the same axes, sketch the graphs of 1, and 4, Show clearly the coordinates, of all points at which the graphs meet the coordinate axes. (marks) The lines f, and fy intersect at the point 4. b Calculate, as exact fractions, the coordinates of 4. (marks) ¢ Find an equation of the line through 4 which is perpendicular to 4. Give your answer in the form ax + by + ¢= 0, where a, b and ¢ are integers. (marks) The points 4 and B have coordinates (4, 6) and (12, 2) respectively. ‘The straight line /, passes through 4 and B. a Find an equation for j in the form ax + by += 0, where a, band care integers. (3 marks) The straight line /) passes through the origin and has gradient b Write down an equation for (1mark) ‘The lines J, and f intersect at the point € ¢ Find the coordinates of C. (2 marks) Show that the lines OA and OC are perpendicular, where O is the origi (2 marks) Work out the lengths of O4 and OC. Write your answers in the form &vT3. (marks) f Hence calculate the area of AOAC. (marks) a Use the distance formula to find the distance between (4a, a) and (3a, 2a). Hence find the distance between the following pairs of points: b (4,1) and (-3, 2) © (12, 3) and (-9, 6) a ( 10) S) and (15, TT] er arsan} SASL} 17 Ais the point (1, 5). Let (x, y) be any point on the line y = 3x. a Write an equation in terms of » for the distance between (x, ») and (I, 5), (3marks) b Find the coordinates of the two points, B and C, on the line y = 3x which are a distance of 74 from (-1, 5). (3marks) ¢ Find the equation of the line J, that is perpendicular to y = 3x and goes through the point (-1, 5). (2marks) Find the coordinates of the point of intersection between J, and y = 3x (2 marks) € Find the area of triangle ABC (marks) Ces 1 Find the area ofthe triangle with vertices A(-2, -2), BU3,8) and C\ 2 Atriangle has vertices AG, 8), 19,9) and C15, 2) as shown in the diagram, The line fis perpendicular to AB and passes through C. The line 4 is perpendicular to BC and passes through A. The line fis perpendicular to AC and passes through B. Show that the lines J, and J, meet at point and find the coordinates of that point. 3 Atriangle has vertices (0,0), ta, b) and Cie, 0) as shown in the diagram, The line is perpendicular to 4B and passes through C. ‘The line is perpendicular to BC and passes through A. ‘The line is perpendicular to AC and passes through B. Find the coordinates ofthe point of intersection off, fs and fy Senet uaa sy Crary een 1 The gradient m of the line joining the point with y coordinates (x,, y;) to the point with coordinates be ve) (xe) can be calculated using the formula ye-3i 2 © The equation of a straight line can be written in the form VEMe+G ya Ee where m is the gradient and (0, ¢) is the y-intercept. © The equation of a straight line can also be written in the form 4 i axtby+e=0, where a, b and care integers 3 The equation of a line with gradient m that passes through the point with coordinates (xy) can be written as.» = yy =m(x— x). 4 Parallel lines have the same gradient. 5 Ifaline has a gradient ma line perpendicular to it has a gradient of — 2. 6 If two lines are perpendicular, the product of their gradients is —1. 7 Youcan find the distance d between (x;, 1) and (x2, y2) by using the formula d= lx — 4? + Or =. 8 The point of intersection of two lines can be found using simultaneous equations. WARREN SK N Peed ‘After completing this chapter you should be able to: ‘© Use the cosine rule to find a missing side or angle > pages 105-210 ‘© Use the sine rule to finda missing side or angle > pages 110-116 ala alalal, ote ear aitangeane sina driatel : formula + pages 116-118 | @ Solve problems involving triangles ~ pages 118-122 | © Sketch the graphs of the sine, cosine and tangent | functions > pages 129-325 Sketch simple transformations of these graphs > pages 125-129 soa daa kad 1 Use trigonometry to find the lengths of the marked sides. a b ss Ia € International GCSE Mathematics 2 Find the sizes of the angles marked. 2 Sia B Sem 620m pag Zacm S Trigonometry in both two and three 4 International G¢SE Mathematics dimensions is used by surveyors to work ‘ ut distances and areas when planning building projects. You will also use trigonometry when working with vector quantities in mechanics, TRIGONOMETRIC rai) 105 @ The cosine rule ‘The cosine rule can be used to work out missing sides or angles in triangles. = This version of the cosine rule is used to find a missing side if you know two sides and the angle between them: a= b+ 2becosA 4 (ns 7 é You can use the standard trigonometric ratios fr right-angled triangles to prove the cosine rule: ; Fora rightangled triangle: it Ba a ret y “ wv gape adler 005 0 F potenuse naacent opposite 4 Ss = ; Yess TE If you are given all three sides and asked to find an angle, the cosine rule can be rearranged. + Phe cos A= +e Phecos A=B+e-a Bre-@ 2be You can exchange the letters depending on which angle you want to find. Hence cos A= = This version of the cosine rule is used to find an angle if you know all three sides: Brena Ai es 2be ary aT Ui Calculate the length of side AB in triangle ABC in which AC = 6.5em, BC =8.7em and ZACB = 100°, 2 =a? +b? ~ 2ab cos C = 87 4652-2 x 87 = 7569 + 42.25 ~ (19639...) 13757. Soe= 1729, So AB= M17 cm (3 GEE 2) EE oss Find the size of the smallest angle in a triangle whose sides have lengths 3em, Sem and 6cm. . < Som 2 oa enone cn 8 “2xSRG Bee ay Cats) ald Coastguard station Bis km, on a bearing of 060°, from coastguard station A. A ship Cis 4.8 km, on a bearing of 018°, away from 4, Calculate how far Cis from B. Gece Ifo diagram is given with a question, you should draw one carefully. Double-check that the information given in the question matches your sketch. a? = b? + 6 ~ 2he cos A 4B? +B 2x 48x BX cos 42% Cis 5.47 km from coastguard station B. 29.966. a= 5.47 (3 51) In AABC, AB =.xom, BC=(x-+2)em, AC =Somand ZABC = 60°. Find the value of x. e+ 2am +e — 2ae cos B (r+ 2% + x2 - 2ulx + 2) cos GO” So 25 = Ox? 44x44 —x?- 2x aii} TRIGONOMETRIC RATIOS GE) Ex Give answers to 3 significant figures, where appropriate. 1 Ineach of the following triangles, calculate the length of the mising side; b B Sem ‘ ase Tem 5 Taem Ic Ssem = € B Sem, ‘10cm, AN 2. In the following triangles, calculate the size of the angle marked x: 2 b 8 c B 25cm, dom Sem Jom 350m 4 4 © c Toom Pie c 4 25em, Siem Tem 3. A plane flies from airport A on a bearing of 040° for 120km and then on a bearing of 130° for 150 km, Calculate the distance of the plane from the airport. ® 13 in AABC, AB= Bee a aU) rai) 4 From a point 4, a boat sails due north for 7km to B. The boat leaves B and moves ona bearing of 100° for 10 km until it reaches C. Calculate the distance of C from A. 5 A helicopter flies on a bearing of 080° from 4 to B, where AB = 50km. It then flies for 60 km to a point C. Given that Cis 80km from 4, calculate the bearing of C from A. 6 The distance from the tee, T, to the flag, F, on a particular hole of a golf course is 494 yards. A golfer’s tee shot travels 220 yards and lands at the point S, where 2S7F = 22°. Calculate how far the ball is from the flag, 7 Show that cos 4 em, Sem itm ¢ 8 Show that cos P = Q_tm PF Sem so 9 In AABC, AB = Sem, BC = 6em and AC = Wem, Calculate the size of the smallest angle. 10 In ABC, AB=9.3em, BC=6.2em and AC=12.7em. Calculate the size of the largest angle 11 The lengths of the sides of a triangle are in the ratio 2: 3:4, Calculate the size of the largest angle 12 In AABC, AB 3)em, BC= (x + 3)em, AC= Bem and ZBAC= 60". Use the cosine rule to find the value of x: ‘em, BC= (x = 4)em, AC= lem and ZBAC = 60". Calculate the value of x. ® 14 In AABC, AB = (5 ~ xem, BC= (4+ xem, ZABC = 120° and AC= yom. a Show that y? b Use the method of completing the square to find the minimum value of 3°, and give the value of «for which this occurs, -x+6l 110 CHAPTER 6 TRIGONOMETRIC RATIOS ©® 15 In ABC, AB= xem, BC= Sem, AC=(10~ xem. 4x=15 a Show that cos ZABC = 5 1 work out the value of »: bb Given that cos 2ABC=—1, ©® 16 A farmer has a field in the shape of a quadrilateral as shown. 2 ism ie Se You will have to use the cosine rule twice. foam Copy the diagram and write any angles cr lengths you work out on your copy. 75m A Tim ‘The angle between fences 4B and AD is 74°, Find the angle between fences BC and CD. GB) 17 The diagram shows three cargo ships, 4, B and C, which are in the same horizontal plane. Ship B is 50km due north of ship A and ship Cis 70km from ship 4. The bearing of C from A is 020°. a Calculate the distance between ships B and C, in kilometres to 3s. (G marks) “akin b Calculate the bearing of ship C from ship B. (4marks) Sk ©) The sine rule The sine rule can be used to work out missing sides or angles in triangles. This version of the sine rule is used to find the length 4 of a missing side: abe 7 sind sinB~ sin You can use the standard trigonometric ratios for right-angled triangles to prove the sine rule: < 4 a | iam h GED cxpiore the sine rule using & sinB=4sh=asinB technology. Bee a aU) rai) at ee and sin A= Po h= bind So asin B=bsin A ab sin A Sin B Ina similar way, by drawing the perpendicular from B to the side AC, you can show that: Sin A Sin E °° Sin A 3in B~ Sin © | This version of the sine rule is used to find a missing angle: sind sin B inc a b a In AABC, AB = 8em, ZBAC = 30° and ZBCA = 40°. Find BC. | Seana Example @J skis In AABC, AB = 3.8m, BC .2em and ZBAC = 35°. Find ZABC. B 1 112| CHAPTER 6 TRIGONOMETRIC RATIOS C= 24781 B= 120° (8 us Give answers to 3 significant figures, where appropriate. 1 Incach of parts a to d, the given values refer to the general triangle. 2 a Given that a= Sem, 4 = 30°, B= 72°, find b. “ b Given that a = 240m, A= 110°, C= 22°, find ¢ ¢ c ¢ Given that b= 14.7om, = 30°, C: Given that c= 9.8em, B= 68.4", 4 ° 2 Ineach of the following triangles calculate the values of x and a a b vem VW) Le) 25a sn Ts LN | a — In parts eand d, ip start by finding the size of the third angle. i= aia} TRIGONOMETRIC RATIOS 2. Work out the possible sizes of x in the Following triangles. a 8 b c c c 40cm so B B 30cm c 65cm Sem ea ‘ V 4 4 3 A fenced triangular plot of ground has area 1200m*, The fences along the two smaller sides are 60m and 80m respectively and the angle between them is @. Show that = 150°, and work out the total length of fencing. ©® 4 Intriangle 48C, BC= (x +2)em, 4 AC = xem and ZBCA = 150°. Given that the area of the triangle Sa is Sem?, work out the value of x, giving your answer to 3 significant figures. widen 5 In APOR, PO=(x+2)em, PR=(5 The area of the triangle is em xyem and ZQPR = 30”, a Show that 4 = 4(10 + 3x-2°) (B marks) b Use the method of completing the square, or otherwise, to find the maximum value of 4, and give the corresponding value of (Amarks) 6 In AABC, AB= xem, AC=(5+x)em Problem-solving and ZBAC = 150°. Given that the area of the : Shem? x tepresents a length soit must be positive. triangle is 3}em?, a show that x satisfies the equation x2 + Sx-15=0 (3 marks) b calculate the value of «x, giving your answer to 3 significant figures, (3marks) ©) solving triangle problems You can solve problems involving triangles by using the sine and cosine rules along with Pythagoras’ theorem and standard right-angled triangle trigonometry. If some of the triangles are right-angled, try to use basic trigonometry and Pythagoras’ theorem first to work out other information. Hyouencounter a triangle which isnot ightanged, EINER, you will need to decide whether to use the sine rule a Saar or the cosine rule. Generally, use the sine rule when , know one side and an opposite a you are considering two angles and two sides and oo the cosine rule when you are considering three sides Greist ers and one angle ea rai) For questions involving area, check first before using the formula involving sine. + To find an unknown angle given two sides and one opposite angle, use the sine rule. * To find an unknown side given two angles and one opposite side, use the sine rule. + To find an unknown angle given all three sides, use the cosine rule. + To find an unknown side given two sides and the angle between them, use the cosine rule. + To find the area given two sides and the angle between them, use Area = 3h sin C. Example rons sown ‘The diagram shows the locations of four mobile phone masts in a field. BC= 75m, CD = 80m, B you can use Area =! x base x height, angle BCD = 55° and angle ADC = 140°. In order that the masts do not interfere with each 75m other, they must be at least 70m apart, Given that 4 is the minimum distance from D, find: a (> a the distance from A to B b the angle BAD UW ie © the area enclosed by the four masts. B cures 75 Split the diagram into two triangles. Use the information in triangle BCD to work out the length BD. You are using three sides and c ‘one angle so use the eosine rule. 4 BD? = 75? + 80? ~ 2(75)(60)cos 55" enpc= 38984 oe sat — Eee rT 120 CHAPTER 6 TRIGONOMETRIC RATIOS AB? = AD? + BD* ~ 2(AD\|BD)cosZBDA AB? = 70° + 71.708..2 = 2(70)(71.708....cos (61045...) _cinZBDA AB® = 847955. 92.084, 92.1m (3 28) AB = BA7955. 045...) x 71.706... 92.084. = 0769 230-1 etapa ZBAD = 5028, rea BCD + area BDA © Area ABCD Area ABCD = 3(BC)(CD)sin ZBCD + MAB) AD)sin BAD Area ABCD = 1(75{6Orin 55") + 192.084... \70)sin(50.26..°) Area ABCD = 24574... + 2479.2. Area ABCD = 49366... = 4940? (3 54) TENSE) TED rencu sms Try to use the most efficient method, and give answers to 3 significant figures. 1 Ineach triangle below find the values of x, y and 2, b 4 Bee a aU) rai) Qe °o In ABC, calculate the size of the remaining angles, the lengths of the third side and the area of the triangle given that a ABAC=40°, AB =8.5em and BC = 10.2cm b AACB= 110°, AC= 4.9m and BC=6.8em A hiker walks due north from 4 and after 8km reaches B. She then walks a further 8 km on a bearing of 120° to C, Work out a the distance from A to Cand b the bearing of C from 4. A helicopter flies on a bearing of 200° from A to B, where AB = 70km. It then flies on a bearing of 150° from B to C, where Cis due south of 4. Work out the distance of C from A. ‘Two radar stations 4 and Bare 16km apart and 4 is due north of B. A ship is known to be on a bearing of 150° from 4 and 10km from B. Show that this information gives two positions for the ship, and calculate the distance between these two pos Find «in each of the following diagrams: von D Sem In SABC, AB=4em, BC=(x + 2)em and AC= Tem. a Explain how you know that 1 0, write down athe value of p (mark) b the coordinates of D (1 mark) © the smallest value of & (1mark) the value of g. (mark) Consider the Function fx) = sin px, p © R, 0-5 x = 360°, ‘The closest point to the origin that the graph of f(x) crosses the x-axis has x-coordinate 36°. a Determine the value of p and sketch the graph of (marks) b Write down the period of f(x). (1mark) The graph shows y= sin @, 0 < @ = 360°, with one value of 0(0 = a) marked on the axis. a Copy the graph and mark on the é-axis the positions of 180° ~ a, 180° + a, and 360° - a, b Verify that: sin cr= sin (180° in (180° + a) = ~sin (360° ~ a). a Sketch on separate sets of axes the graphs of » = cos, 0 = 0 = 360°, and Jy =tan 8, 0 < @-< 360°, and on each é-axis mark the point (a, 0) as in question 16. b Verify that: i cosa ii tano= cos (180° ~ a) = -c08(180° + 2) = cos(360° ~ a) tan (180° — a) = tan (180° + a) = ~tan (360° — 0) A series of sand dunes has a cross-section which can be modelled using a sine curve of the form y = sin (60x°) where x is the length of the series of dunes in metres, a Draw the graph of y= sin (603°) for 0 radians. 3 In the diagram, AD and BC are ares of circles Od = OD = rem, AB = DC= 8em and ZBOC= 0 radians, 4 Given that the area of the shaded region is 48cm? cea, show that * oy (4 marks) e b Given also that r = 100, calculate the perimeter < of the shaded region. (6 marks) A sector of a cirele of radius 28cm has perimeter Pem and area Acm* Given that A = 4P, find the value of P. ‘The diagram shows a triangular plot of land. The sides 4B, BC and CA have lengths 12m, 14m and 10m respectively. The lawn isa sector of a circle, centre 4 and radius 6m. a Show that BAC = 1.37 radians, correct to 3 significant figures. b Calculate the area of the flowerbed. The diagram shows OPQ, a sector of a circle with centre O, 2 i 0 radius 10cm, with ZPOQ = 0.3 radians, The point Ris on OQ such that the ratio OR: RQ is 1:3. The region S, shown shaded in the diagram, is bounded by fem OR, RP and the are PO. Find: P a the perimeter of S, giving your answer to 3 significant figures (6 marks) b the area of S, giving your answer to 3 significant figures, (marks) ‘The diagram shows the sector OAB of a circle with centre O, radius 12cm and angle 1.2 radians. The line AC is a tangent to the circle with centre O, and OBC is a a PN straight line. The region R is bounded by the arc 4B and the lines AC and CB. NS a Find the area of R, giving your answer to 2 decimal places, (8 marks) ‘The line BD is parallel to AC b Find the perimeter of DAB. (5 marks) RADIANS @®u Challenge Find an expression forthe area ofa sector of a circle with radius r and arc length I ated ‘The diagram shows two intersecting sectors: ABD, with radius Sem and angle 1.2 radians, and CBD, with radius 12cm. Find the area of the overlapping section. CEE 7) EEE) excornersncnon ®1 &® 2 Triangle ABC is such that AB= Sem, AC An are of a circle, centre 4 and radius Sem, a State, in radians, the value of BAC. b Calculate the area of the region enclosed by BC, DC and the are BD. Ocm and ZABC= 90", cuts AC at D. ‘The diagram shows the triangle OCD with OC= OD = 17cm and CD = 30em. The midpoint of CD is Mf. A semicircular em Vem are Aj, with centre M is drawn, with CD as diameter. A circular are 4 with centre O and radius 17em, is drawn from C to D. The shaded region R is bounded by the ares © 4 7 A, and A, Calculate, giving answers to 2 decimal places: a the area of the triangle OCD (4 marks) b the area of the shaded region R. (marks) 4, The diagram shows a circle, centre O, of radius Gem, The points 4 and Bare on the circumference of the circle. ‘The area of the shaded major sector is 80cm? Given that AOB = 0 radians, where 0 <0 < x, calculate: 1 athe value, to 3 decimal places, of @ (B marks) the length in cm, to 2 decimal places, of the minor are AB. (2 marks) a The diagram shows a sector O4B of a circle, centre O and radius 4 rem, The length of the are AB is pem and ZAOBis 0 radians. a Find 0 in terms ofp and r marks) b Deduce that the area of the sectoris } prem? 2marks) Given that r= 4.7 and p = 5.3, where each has been measured to I decimal place, find, giving your answer to 3 decimal places re € the least possible value of the area of the sector (2marks) f 4 the range of possible values of 8. (marks) 0: em Sema PON ®s5 ‘The diagram shows a circle centre O and radius Sem, The length of the minor are AB is 6.4em, a Calculate, in radians, the size of the acute angle < AOB. (2 marks) The area of the minor sector AOB is Rycm? and the area of the shaded major sector is Rem? b Calculate the value of Ry (2 marks) © Caleulate Rj: Ry in the form 1:p, giving the value of p to 3 significant figures. (3 marks) oo The diagrams show the cross-sections of two 7 drawer handles. Shape Xs a rectangle ABCD Joined to a semicircle with BC as diameter ey ‘The length AB = dem and BC = 2dem Shape Y isa sector OPQ of a circle with 2 2 centre O and radius 2dem, Angle POQ is Oradians. Given that the areas of shapes Vand Yare @ equal, prove that @ = 1+ re (5 marks) Using this value of @, and given that d=3. Shape X Shope ¥ find in terms of x: b the perimeter of shape Y (G marks) © the perimeter of shape Y. (@ marks) 4 Hence find the difference, in mm, between the perimeters of shapes X and ¥. (mark) p ‘The diagram shows a circle with centre O and radius 6em, ‘The chord PQ divides the circle into a minor segment Rj of area A\cm? and a major segment R; of area Aem?, The chord PQ subtends an angle @ radians at O. a Show that A) = 18(@ — sin), (2 marks) b Given that 4, = 34,, show that sin @ (Amarks) @ Triangle ABC has AB= 9m, BC = 10cm and C4 = Sem, 4 A circle, centre 4 and radius 3em, intersects 4B and AC at P and Q respectively, as shown in the diagram, / er a Show that, to 3 decimal places, Sent ZBAC = 1.504 radians, (2 marks) b Calculate: B i thearea, in cm?, of the sector APO € ‘oom the area, of the shaded region BPQC the perimeter, in cm, of the shaded region BPOC. (8 marks) RADIANS &® » Cd ‘The diagram shows the sector OB of a circle of radius rem. The area of the sector is 1Sem? and ZAOB = 1.5 radians, 4 a Prove that r= 2V5, (2 marks) b Find, in om, the perimeter of the sector O4B. marks) ) / ‘The segment R, shaded in the diagram, is enclosed by the are AB and the straight line AB. © Calculate, to 3 decimal places, the area of R. (2 marks) ° ‘The shape of a badge is a sector ABC of a circle with centre A and radius 4B, as shown in the diagram. The triangle ABC is a € equilateral and has perpendicular height 3em. a Find, in surd form, the length of AB (2 marks) b Find, in terms of =, the area of the badge. (2 marks) € Prove that the perimeter of the badge is=*(x-+ 6)em. (4 marks) There is a straight path of length 70m from the point 4 to the point B. The points are joined also by a railway track in the form of an are of the circle whose centre is Cand whose radius is 44m, as shown in the diagram, a Show that the size, to 2 decimal places, of ZACBis 1.84 radians, (2 marks) b Calculate: the length of the railway track ithe shortest distance from Cto the path iii the area of the region bounded by the railway track and the path. (marks) ‘The diagram shows the cross-section ABCD of a glass prism. Adem dD AD = BC = 4em and both are at right angles to DC. ABis the are of a circle, centre O and radius Gem. Given that 2AOB = 29 radians, and that the perimeter of the cross-section is 2(7 + xem, a show that (20 + 2sin@ — 1 b verify that tem € find the area of the cross-section. ‘Two circles C, and C;, both of radius 12cm, have centres O, and O, respectively. O, lies on the circumference of Cy; O, lies on the circumference of C,, The circles intersect at A and B, and enclose the region R. a Show that 240,B ==. b Hence write down, in terms of 1, the perimeter of R. ¢ Find the area of R, giving your answer to 3 significant figures. rd Smale PON 14 A teacher asks a student to find the area of the following sector. ‘The attempt is shown below ‘rea = 310 3x 3?x 50 = 225m? a Identify the mistake made by the student. b Calculate the correct area of the sector. Two circles of radii 10cm and 8cm respectively intersect at points 4 and B such that the length of line 4B is 14cm, a For each separate circle, calculate in radians to 3 d.p. the angle subtended at the centre by the arc 4B. b Hence, or otherwise, calculate to 1 dip. the area of the shaded region R, bounded by the two radii ofthe larger circle and the arc of the smaller circle, (1 mark) (marks) RADIANS ead Ben 1 + 2rradians=360° + rradians=180° © L radian 2 Ifthe arc AB has length r then 2 AOB=1 radian (or 1 rad or 14) 3 + 60°=F radians + 90° = radians + 180° = radians + 360° = 2m radians Aradian is the angle subtended at the centre ofa circle by an arc whose length is equal to that of the radius of the circle. To find the arc length /of a sector of a circle, use the formula /= r9, where ris the radius of the circle and is the angle, in radians, contained by the sector. To find the area 4 of a sector of a circle, use the formula A = 4770, where ris the radius of the circle and is the angle, in radians, contained by the sector The area of a segment in a circle of radius ris A=12(0 - sind) oT) 8) MEE Learning objectives rorer er) leting this chapter you should © Find the derivative, f"(x) or —, of a simple function > pages 154-163 © Use the derivative to solve problems involving gradients, tangents and normals > pages 163-165 ay © Find the second derivative, F(x) or, of a - d simple function «Section 5.1 1 each of these expressions in the form x" where ris a positive or negative real number. : « Sections 1.1, 14 Find the equati e straight line that passe Differentiation is part of calculus, one of pee the most. in mathem nd (6,1) b (3,7) and (9,4) You will use differentiation in mechanics « (10,5) and (-2, 8) «Section 5.2 to model 5 Find the equation of the perpendicular tot! and acceleration, y= 2x ~5 at the point (2, -1) «Section 5.3 eu

You might also like